Этого треда уже нет.
Это копия, сохраненная 3 ноября 2016 года.

Скачать тред: только с превью, с превью и прикрепленными файлами.
Второй вариант может долго скачиваться. Файлы будут только в живых или недавно утонувших тредах. Подробнее

Если вам полезен архив М.Двача, пожертвуйте на оплату сервера.
78 Кб, 640x480
Математика, тред 50 #372131 В конец треда | Веб
Далеко-далеко в самом гладком из гладких многообразий на самой его границе жила дифференциальная форма. И была та форма замкнута и имела такой скверный характер, что не было у той формы ни друзей, ни близких, и нельзя было встретить никого даже в самой малой эпсилон окрестности этой границы.
Но однажды в том краю появился мудрый и удовлетворяющий всем аксиомам отделимости Гасан Заде. И пошел он сразу на самый край этого многообразия и встретил там эту самую форму. И начала форма ругаться на него, почэму он ходит по многообразию, пачиму баламутит метрический тензор. На что Гасан Задэ невозмутимо спросил: "Форма, всю жизнь ты живешь на этой границе, а знаешь ли ты каков твой интеграл по ней?
И задумалась форма, и не было у нее мыслей кроме этой, бросила форма все сваи занятия, перестала пить и есть, потиряла покой и сон. Семь дней и восемь ночей пыталась форма проинтегрироваться по контуру, но ничего у нее нэ получалось: пыталась форма интегрировать в лоб, пыталась перейти к другим координатам, а когда савсем отчаялась, устремила к бисканечности интегральные суммы, но патирпела полный провал, поскольку учил ее интегрировать велеречивый, но не очень ученый хан Агахан.
И пришла форма исхудавшая до такой степени, что было видно у нее внешнее произведение, к Гасану Заде и протянула к нему в мольбе координаты свои: "О мудрый и метризуемый Гасан Заде, помоги мне проинтегрироваться по контуру, ибо не в силах я терпеть дальше такие унижения!" И отвечал ей невозмутимый и сепарабельный Гасан Заде: "Интеграл твой равен нулю и было известно мне еще до того, как подошел к краю, поскольку одинока ты и нет у тебя ни друзей ни образа при дифференцировании". "Ну и что с того того!” - возопила форма. "А то, что известна мне формула Стокса и ясна она мне как первый луч весеннего солнца взошедшего над алгебраически замкнутым полем"
И услышав слова такие не выдержала форма, подошла к границе и прыгнула с нее в самые пучины бесконечномерных топологий, так что не сыскать ее теперь, поскольку топологий этих несчислимое множество и занимается ими совсем другая наука.
Так поднимем же точные последовательности за то, чтобы умели мы считать когомологии, невзирая ни на гомотопический тип, ни на символы Кристофеля, что причиняют столько неприятностей и по сей день.

---

Этот тред посвящён pure mathematics. Здесь из нескольких разных доказательств выбирают самое концептуально правильное, доказывают задачи из листочков для первого курса НМУ, а также читают научные статьи (пытаются).

Если вы хотите задать вопрос типа "как мне взять этот интеграл", это не сюда. Вам нужен тред "для начинающих", он плавает неподалёку.

Предыдущий: >>355215 (OP) (OP)
Архивы #2 #372132
1. https://arhivach.org/thread/18638/
2. https://arhivach.org/thread/27246/
3. https://arhivach.org/thread/27696/
4. https://arhivach.org/thread/38709/
5. https://arhivach.org/thread/46502/
6. https://arhivach.org/thread/48852/
7. https://arhivach.org/thread/52165/
8. https://arhivach.org/thread/56479/
9. https://arhivach.org/thread/63306/
10. https://arhivach.org/thread/70618/
11. https://arhivach.org/thread/74342/
12. https://arhivach.org/thread/74341/
13v1. https://arhivach.org/thread/76561/
13v2. https://arhivach.org/thread/92428/
14. https://arhivach.org/thread/78408/
15. https://arhivach.org/thread/79152/
16. https://arhivach.org/thread/82499/
17. https://arhivach.org/thread/92427/
18. https://arhivach.org/thread/84722/
19. https://arhivach.org/thread/87923/
20. https://arhivach.org/thread/91329/
21. http://arhivach.org/thread/93067/
22. https://arhivach.org/thread/94240/
23. https://arhivach.org/thread/95680/
24. https://arhivach.org/thread/96720/
25. https://arhivach.org/thread/99481/
26. https://arhivach.org/thread/100880/
27. https://arhivach.org/thread/101335/
28. http://arhivach.org/thread/106743/
29. https://arhivach.org/thread/109198/
30. https://arhivach.org/thread/114111/
31. https://arhivach.org/thread/116099/
32. https://arhivach.org/thread/118093/
33v1. https://arhivach.org/thread/122613/
33v2. https://arhivach.org/thread/122615/
34. https://arhivach.org/thread/123717/
35. https://arhivach.org/thread/128822/
36. https://arhivach.org/thread/129039/
37. https://arhivach.org/thread/131462/
38. https://arhivach.org/thread/138362/
39. https://arhivach.org/thread/138429
40. http://arhivach.org/thread/140404/
41. https://arhivach.org/thread/142386/
42. https://arhivach.org/thread/145879/
43. https://arhivach.org/thread/146833/
44. https://arhivach.org/thread/152600/
45. https://arhivach.org/thread/153157/
46. https://arhivach.org/thread/156244/
47. https://arhivach.org/thread/159628/
48. https://arhivach.org/thread/165872/
49. https://arhivach.org/thread/183576/
Архивы #2 #372132
1. https://arhivach.org/thread/18638/
2. https://arhivach.org/thread/27246/
3. https://arhivach.org/thread/27696/
4. https://arhivach.org/thread/38709/
5. https://arhivach.org/thread/46502/
6. https://arhivach.org/thread/48852/
7. https://arhivach.org/thread/52165/
8. https://arhivach.org/thread/56479/
9. https://arhivach.org/thread/63306/
10. https://arhivach.org/thread/70618/
11. https://arhivach.org/thread/74342/
12. https://arhivach.org/thread/74341/
13v1. https://arhivach.org/thread/76561/
13v2. https://arhivach.org/thread/92428/
14. https://arhivach.org/thread/78408/
15. https://arhivach.org/thread/79152/
16. https://arhivach.org/thread/82499/
17. https://arhivach.org/thread/92427/
18. https://arhivach.org/thread/84722/
19. https://arhivach.org/thread/87923/
20. https://arhivach.org/thread/91329/
21. http://arhivach.org/thread/93067/
22. https://arhivach.org/thread/94240/
23. https://arhivach.org/thread/95680/
24. https://arhivach.org/thread/96720/
25. https://arhivach.org/thread/99481/
26. https://arhivach.org/thread/100880/
27. https://arhivach.org/thread/101335/
28. http://arhivach.org/thread/106743/
29. https://arhivach.org/thread/109198/
30. https://arhivach.org/thread/114111/
31. https://arhivach.org/thread/116099/
32. https://arhivach.org/thread/118093/
33v1. https://arhivach.org/thread/122613/
33v2. https://arhivach.org/thread/122615/
34. https://arhivach.org/thread/123717/
35. https://arhivach.org/thread/128822/
36. https://arhivach.org/thread/129039/
37. https://arhivach.org/thread/131462/
38. https://arhivach.org/thread/138362/
39. https://arhivach.org/thread/138429
40. http://arhivach.org/thread/140404/
41. https://arhivach.org/thread/142386/
42. https://arhivach.org/thread/145879/
43. https://arhivach.org/thread/146833/
44. https://arhivach.org/thread/152600/
45. https://arhivach.org/thread/153157/
46. https://arhivach.org/thread/156244/
47. https://arhivach.org/thread/159628/
48. https://arhivach.org/thread/165872/
49. https://arhivach.org/thread/183576/
#3 #372133
>>372131 (OP)
А настоящий предыдущий там: >>368293 (OP)
#4 #372219
>>372131 (OP)
Прошлая картинка лучше была.
sage #5 #372280
И сразу сажи аниму-пидорам.
#6 #372341
>>372131 (OP)

>потиряла


Ну йобана.
>>372343
#7 #372343
>>372341
Ошибки - это не баг, это специально.
>>372349
#8 #372349
>>372343
Ты случайно не танкист? А то там такой же дебил был писавший ровно тоже самое.
#9 #372354
Не взлетает что-то тематическое обсуждение. А я пока напоминаю вам, что действительные числа не нужны.
>>372366
#10 #372366
>>372354
А что вообще можно тут обсуждать? Я вот просто читаю книги. У меня нет причин привлекать как-то других людей. Возникающие вопросы прекрасно разрешаются с помощью гугла.

Могу задать задачки, если хотите.

Семейство подмножеств множества X называется сигма-алгеброй над X, если объединение его счётного подсемейства является его элементом, пересечение его счётного подсемейства является его элементом, дополнение его элемента является его элементом.

Пусть X - множество, пусть M - семейство его подмножеств.
1. Докажите, что существует сигма-алгебра, являющаяся надмножеством M.
2. Докажите, что пересечение всех сигма-алгебр, являющихся надмножествами M, снова сигма-алгебра.
Таким образом, каждое семейство подмножеств X содержится в некоей наименьшей по включению сигма-алгебре.

Пусть X - вещественная прямая со стандартной топологией. Наименьшая сигма-алгебра, содержащая все открытые множества, называется борелевской.

3. Докажите, что мощность борелевской сигма-алгебры равна континууму.
Указание. С помощью трансфинитной рекурсии определите бесконечно много классов таких, что элементами последующего являются счётные объединения и пересечения элементов предыдущего. Начните с класса всех отрезков. Доведите трансфинитное построение до первого несчётного ординала.
#11 #372368
Кто бы что ни говорил. А мне вот доставляли споры о натуральных числах, с перебрасыванием на философские статейки, темы на MO и SE и всем таким. Было весело.

Твои задачки ведь совсем упражнения, давай чего-нибудь полуолимпиадного или на глубокое понимание. В идеале ещё, чтобы не гуглилось. Ну или я могу дать:

Найти все неизоморфные плотные линейные порядки порядки без концов, такие, что каждый начальный отрезок счётен.

1,2 - конструкция порождённой сигма-алгебры.
3 - борелевская иерархия множеств.
>>372490
#12 #372369
>>372131 (OP)
Что ззначает производная?Приращение функции относительно переменной, да.
Вот функция икс квадрат. Её производная 2 икс.
При икс пять функция равна 25, производная 10
при шесть 36, 12, при семи 49, 14.
Вот в каком месте это приращение? Относительно чего?
>>372386
#13 #372371
1. Любая абелева группа над алгебраически замкнутым полем имеет только одномерные неприводимые представления. Верно ли обратное, что любая группа, у которой только одномерные неприводимые представления - абелева? Верно-ли для конечных?
2. Есть примеры известных и использующихся в науке функторов, которые точны слева/справа, но не являются при этом правыми/левыми сопряжёнными к каким-то?
>>372374
#14 #372372
>>372366
Ну давай, зубри, зубри промытыш, свои бессмысленные сигма-алгебры
74 Кб, 879x920
21 Кб, 746x259
62 Кб, 746x849
#15 #372373
>>372366
1. Множество всех подмножеств X.
2. Тривиальная проверка.
3. Ясно, что борелевская сигма-алгебра несчетна, потому что она содержит все одноточечные множества. Мощности несчетных множеств между собой я различать не собираюсь, это эзотерико-мистическая хуйня для дрочил.

Вот лучше найдите мне нормальное доказательство формулы Тейлора. По запросу "intuition for taylor's series" ниче хорошего нет.
У Львовского нашел интуитивное объяснение, почему коэффициенты такие (пик 1), но получение остаточного члена какое-то совсем трюковое (пик 2 и 3). Кто-нибудь знает попроще вывод?
>>372375>>372380
#16 #372374
>>372371
Повторяю, нахуя тебе это знать?
#17 #372375
>>372373
Если множество борелевских множеств континуально, то, так как множество всех подмножеств R более чем континуально, существуют неборелевские множества, что довольно-таки забавно.
>>372378
#18 #372376
>>372374
Чтоб у тебя, картофана, бомбануло от того, что люди развлекаются, а ты дрочишь свой картофан.
>>372377
#19 #372377
>>372374
Чтобы на двачах выебываться.

>>372376
У картофана есть возможность устроиться на нормальную работу, а ты так и будешь у мамки на шее сидеть.
#20 #372378
>>372375

> что довольно-таки забавно.


Почему? Для этого и придумали борелевские сигма-алгебры: чтобы не рассматривать патологические множества.
>>372379
#21 #372379
>>372378
Потому что априори совершенно не очевидно существование множеств, которые не могут быть получены из интервалов с помощью счётного количества применений операций объединения, пересечения и взятия дополнения.
>>372381>>372383
#22 #372380
>>372373
Наименее трюковое доказательство тут
http://math.stackexchange.com/questions/481661/simplest-proof-of-taylors-theorem
как по мне.

>это эзотерико-мистическая хуйня для дрочил.


Это единственное, чем я заниматься собираюсь!
>>372384>>372388
#23 #372381
>>372379
Так это маняматики понапридумывали своих аксиом выбора и верят в существование таких множеств. Нормальные люди используют аксиому конструктивности и получают вполне интуитивный и концептуально верный для анализа результат - таких множеств не существует.
>>372383
#24 #372383
>>372381
>>372379
Ну это несерьезно даже.
Если у меня есть непустое множество, почему я не могу выбрать из него элемент? Что за бред?
>>372385
#25 #372384
>>372380

> Наименее трюковое доказательство тут


Спасибо, анон, обязательно посмотрю.
>>372388
#26 #372385
>>372383
Из одного непустого - можешь. А из континуального семейства непустых - нет. Потому что само понятие "континуальности" - сказки от маняматиков.
>>372387
#27 #372386
>>372369

>производная?Приращение функции относительно переменной, да.


Производная это не приращение, производная -- это коэффициент линейной части приращения.

Типа есть у тебя функция, f(x). Что такое наличие непрерывности в x? Это когда f(x+dx)-f(x)=o(1). Что такое наличие производной в x? Это когда существует A(x) такой, что f(x+dx)-f(x)=A(x) dx+o(dx). Что такое производная? Как раз этот A(x).

>Вот функция икс квадрат. Её производная 2 икс.


(x+dx)^2 - x^2 = 2 x dx + dx^2 = 2 x dx + o(dx).
Линейная часть приращения 2 x dx, поэтому производная 2x.

>Вот в каком месте это приращение? Относительно чего?


В левой части написано f(x+dx)-f(x), это приращение.
В правой части написано 2 x dx + o(dx). Линейная его часть описывается производной.
>>372392>>372422
#28 #372387
>>372385
Дробных чисел не бывает. Отрицательных чисел не бывает. Нуля не бывает. Чисел более чем 10^20 не бывает.
>>372390
#29 #372388
>>372380
>>372384
(На самом деле нет, нахуй тебя пошлю задротик)))) )
>>372391
#30 #372390
>>372387
Категориями "бывает" и "не бывает" даже в ХХ веке не мыслили. Есть два языковых фреймворка - на аксиоме выбора и на аксиоме конструктивности. Первый усложняет теорию всякими ебучими множествами Витали, заставляет делать постоянно оговорку "пусть f - измерима", и вообще даёт неправильную перспективу и прикладникам, и чистым математикам: будто неизмеримые множества - это реально часто встречающийся эффект, который нужно учитывать, а не лингвистическая конструкция на хуёвом языке; а второй позволяет всего этого избежать и делает теорию в разы стройнее и ставит правильную интуицию (как прикладникам, так и чистым математикам). А свои ебучие демагогические гиперболизации при себе оставь, чёрт.
#31 #372391
>>372388
Относительно точки 5.
>>372392
#32 #372392
>>372394
#33 #372393
>>372390
А что там с неизмеримыми множествами по лебегу, если отрицать аксиомы выбора? Они вообще исчезают?
>>372397
#34 #372394
>>372392
Не понел? Ничем другим кроме описанного производная не является, разве что я не сказал, что она оператор, действующий на приращениях.
#35 #372395
>>372390
Начнём с того, что в конструктивной математике теорема Больцано-Коши не имеет места. Если провести на плоскости прямую, взять в полуплоскостях две разные точки и соединить их непрерывной линией, то у конструктивистов может случиться так, что кривая не будет пересекать прямую. На этом разговор о неправильной интуиции закончим.
>>372396>>372397
#36 #372396
>>372395

>у конструктивистов может случиться так, что кривая не будет пересекать прямую


А почему это кстати? Там ведь никакой аксиомы выбора нет
#37 #372397
>>372393
Нет конечно, принимать отрицание аксиомы выбора - это вообще шиза, так как счётная аксиома выбора постоянно используется в анализе. Они исчезают, если принять аксиому конструктивности.
>>372395
Хуйню говоришь. Ты путаешь аксиому конструктивности с конструктивной математикой типа Маркова. Весь одномерный анализ с аксиомой конструктивности сохраняется точно таким же. По модулю всяких рассуждений, основанных на неравенстве мощностей (которые нахуй не нужны, зачастую, ни прикладникам, ни математикам).
>>372398
105 Кб, 1280x697
#38 #372398
>>372397
Ну давай, докажи теорему Больцано-Коши, ке ке ке.
>>372400
#39 #372399
Ой, блять, прошу прощения всех в теме. Под аксиомой конструктивности я всегда имел в виду аксиому детерменированности.
>>372401
#40 #372400
>>372398
Доказательство полностью перепишется из любого учебника по анализу. Потому что оно нигде не использует несчётный выбор.
>>372401
#41 #372401
>>372399
Почти каждый вывод из аксиомы детерминированности является злоебучкой. С помощью этой аксиомы ты можешь разделить вещественную прямую на более чем континуум попарно непересекающихся непустых множеств. Хотя в ней всего лишь континуум точек.

http://mathoverflow.net/questions/22927/why-worry-about-the-axiom-of-choice/22935#22935

По-прежнему не считаю нужным продолжать разговор о неправильной интуиции.

>>372400
Не-не, ты докажи. Ке ке ке.
#42 #372403
Молодца гуглить умеешь, только заметь, что в той темке нету ни одного факта, на который было бы не поебать аналитику. А размышления о континуумах и о том, на что его можно разбить, а на что нельзя, и как там сравниваются разные бесконечности - нахуй не нужны в анализе, да и вообще неинтересны нормальным людям.

Вас выдрачивали 20 лет, что N и Q равномощны, а N и R - нет, - и вам кажется это очень интуитивным и пиздос каким очевидным. Выдрачивали бы 20 лет, что С можно разбить на более, чем C множеств, то и это бы казалось очевидным и интуитивным. Нормальным людям же заниматься интеллектуальной мастурбацией не нужно.
>>372405
#43 #372404
>>372374
А нахуя тебя спрашивать?
>>372406
#44 #372405
>>372403
Вот когда покажешь мне более чем континуум непересекающихся непустых числовых множеств, тогда и поговорим.
>>372407
#45 #372406
>>372404
Чего?
>>372408
#46 #372407
>>372405
Когда мне покажешь континуум, тогда и поговорим.
>>372409
#47 #372408
>>372406
тебе*
>>372410
#48 #372409
>>372407
Существование R твоя религия не запрещает, насколько я понимаю.
>>372412
#49 #372410
>>372408
Узнать, для чего он просит, чтобы оценить, стоит ли отвечать.
>>372411
#50 #372411
>>372410
А ты знаешь ответ-то?
#51 #372412
>>372409
Моя религия проста: для анализа нужна АD, вопрос о количестве попарно непересекающиеся множеств континуума - не аналитический. Понять, почему АD в анализе лучше, чем AC можно очень просто: взглянуть на длину формулировок и доказательств одних и тех же утверждений в одной теории, и в другой.
>>372417
#52 #372413
Найти все неизоморфные плотные линейные порядки порядки без концов, такие, что каждый начальный отрезок счётен.
Zadacha.
#53 #372417
>>372412
У меня нет причин считать, что AD достаточно для анализа. Никто ещё не взял на себя труд развить анализ на основе этой аксиомы. Скажи, ты можешь доказать, что определения предела по Коши и по Гейне эквивалентны, опираясь на аксиому детерминированности?
>>372424
#54 #372422
>>372386
Что значит

>o(1)


df?
Что такое

>A(x)


Другая функция?

И что получается, производная показывает насколько много вырастет функция при малом увеличении икса?
>>372563
#55 #372424
>>372417
Разумеется, доказательство точно такое же, как и в любом учебнике анализа.
>>372427
#56 #372427
>>372424
В учебнике используется аксиома выбора.
>>372428
#57 #372428
>>372427
Аксиома счётного выбора. Которая следует из AD, разумеется.
>>372430
#58 #372430
>>372428
В произвольных метрических пространствах-то?
>>372431
#59 #372431
>>372436
#60 #372433
>>372390
Эквивалентность пределов по Коши и Гейне докажешь конструктивист мамкин?
>>372455
#61 #372436
>>372431
Ой ли.
#62 #372455
>>372433
Научили дурака умным словам.
#63 #372479
>>372390
Аминь.
#64 #372490
>>372368

>Найти все неизоморфные плотные линейные порядки порядки без концов, такие, что каждый начальный отрезок счётен.



Подскажи, как это решить вообще?
>>372496
#65 #372496
>>372490
Попробуй сначала показать, что любой счётный плотный линейный порядок без концов изоморфен Q.
>>372544
#66 #372544
>>372496
Это-то очевидно.
>>372549
#67 #372549
>>372544
Теперь попробуй придумать пример нужного порядка, неизоморфный Q.
#68 #372563
>>372422

>Что значит o(1)?


https://en.wikipedia.org/wiki/Big_O_notation#Little-o_notation
Функция, стремящаяся к нулю при dx->0.

>И что получается, производная показывает насколько много вырастет функция при малом увеличении икса?


Да, объясняет линейную часть этого роста: f(x+dx)-f(x)=f'(x)dx + o(dx)
120 Кб, 1063x765
739 Кб, 615x831
#69 #372569
Заметил, что по проекции (перспективы) тройки векторов на плоскость можно определить, могли ли они быть равны по длине, в предположении, что они ортогональны.

Но как это сделать просто? Иными словами, как рисовать куб в перспективе, чтобы точно знать, что это куб, что должно выполняться?

Второй вопрос, при фиксированных ортогональных направлениях, легко ли делать параллельный перенос векторов вдоль них, и как?
>>372587
44 Кб, 673x783
#70 #372587
>>372569
Вроде сам понял.
Если кому интересно
Чтобы проверить, что перед тобой не просто прямоуг параллелепипед, а куб, достаточно проверить, что диагонали под 45 градусов. Делить углы пополам в проективке оче просто. Солвед.

Параллельный перенос вдоль прямой можно тоже ухитриться делать через эти 45 градусов, но наверняка можно проще
#71 #372620
>>372131 (OP)
Теория колец и теория множеств. Они как то связанны, вложены друг в дружку, или нет?
>>372623
#72 #372623
>>372620
Да, связаны. Многие факты из теории колец опираются на фундаментальные множественные конструкции.
>>372624
#73 #372624
>>372623
Связанны в каком смысле? У них общее основание или кольца находятся под эгидой теории множеств/наоборот?
>>372625
#74 #372625
>>372624
В присутствии аксиомы выбора получается одна теория колец (с ультрафильтрами и прочим), в присутствии отрицания аксиомы выбора получается совсем другая теория колец, без ультрафильтров вообще говоря.
#75 #372638
>>372131 (OP)
Скажите, p-адичсекие числа есть в программе каких-то факультетов кроме чисто-математических? Как называется дисциплина, в рамках которой они изучаются?
>>372639
#76 #372639
>>372638
Теория чисел.
В глубоком идеале, их должны проходить на факультетах по информационной безопасности. Но всем как всегда.
>>372640
#77 #372640
>>372639
А является ли глубокий идеал полупростым?
#78 #372643
>>372131 (OP)
Вопрос, наверное, странный. Я тот тип, который спрашивал про некоммутативную геометрию, будучи очень далеким от темы. Скажите 1. С какими разделами математики я должен быть знаком для того, чтобы более-менее понимать о чем речь при чтении литературы по некоммутативной геометрии? 2.(навеяно постом чуть выше) Этот минимальный уровень для понимания некоммутативной геометрии можно получить где-то кроме чисто-математических факультетов?
>>372651
#79 #372651
>>372643
Я тот тип, который тебе отвечал
1. C*-алгебры, К-теория, классическая алгебраическая геометрия.
2. Нельзя даже на математических факультетах. Это очень новая ветвь, по которой написано 1.5 книжки и которые развивает 3.5 человека. Найти компетентного человека именно в некоммутативной геометрии - очень сложно.
#80 #372655
>>372651
Т.е. даже если я буду нормально соображать во всем, что ты указал в пункте 1., то без разъяснений компетентного человека я все равно не пойму почти ничего?
#81 #372656
>>372651
Тем не менее, стоит заметить что идти на чисто-математический факультет в любом случае смысла больше чем идти куда-то ещё или не идти никуда - чтобы стать более менее современно образованным математиком и заниматься наукой, заниматься "только некоммутативной геометрией" не получится, узнать надо очень много всего. В конце обучения вообще может оказаться, что некоммутативная геометрия не так уж интересна, а интересно что-то другое совсем.
#82 #372657
>>372655
Не знаю, я сам в ней плохо разбираюсь. Но думаю, что если ты хорошо образован, то книжка Конна может хорошо зайти, и тогда компетентный человек нужен не будет. А может и нет.
А почему именно некоммутативная геометрия? Название понравилось?
#83 #372659
>>372651
>>372655
"C*-алгебры, К-теория, классическая алгебраическая геометрия" а это можно получить где-то кроме чисто математических факультетов?(условно, на ФУПМ пистеха такому учат?)
#84 #372660
>>372659
Ситуация такая, что даже на математических факультетах это не входит в общеобразовательную программу, бывают спецкурсы какие-то, конечно, но это такое.
Вообще с какого-то уровня ты уже сам можешь учиться чисто по учебникам; если ты думаешь, что сидя на лекции ты будешь получать больше фидбэка, чем читая книжку или смотря лекции по ютубу - то ты ошибаешься очень
>>372663
#85 #372661
>>372659

>условно, на ФУПМ пистеха такому учат.


Нет.
#86 #372663
>>372660
Переформулирую: на условном мехмате я смогу получить нужную инфу по этим разделам в виде спецкурсов, или на крайний случай просто помощи конкретного сведущего человека, а на условном ФУПМ смогу, или нет? (ФУПМ привожу как пример сильного, но не чисто-математического факультета)
>>372665
#87 #372664
>>372659
Ок, понял
#88 #372665
>>372663
Очень вряд ли.
>>372666
#89 #372666
>>372665
Последний вопрос. Самый странный, наверное. Хочется иметь как можно более широкий математический кругозор, но при этом учиться не на чисто-математическом факультете(не надо спрашивать почему, просто принят как есть). Есть ли варианты которые существенно лучше ФУПМа?
>>372667>>372978
#90 #372667
>>372666
Ходить в НМУ по вечерам.

Какой лучший факультет прикладной математики - не знаю. Я бы во ВШЭ пошёл, у них денег дохуя - куда-нибудь тебя приткнут после выпуска, если не долбоёб.
#91 #372671
я платонистам покушать принёс:
http://www.runivers.ru/upload/iblock/25d/florensky.pdf
>>372674
92 Кб, 350x467
#92 #372674
>>372671
Лел, Кайло Рен какой-то написал.
>>373066
#93 #372691
Все чаще можно встретить мнение, что мехмат- говно, матфак- сила. Склонен считать, что это скорее правда, чем ложь, но вот в чем вопрос: есть ли тенденция к улучшению ситуации на мехмате?
#94 #372694
>>372691
Есть тенденция к ухудшению ситуации на матфаке.
#95 #372695
>>372691
https://www.youtube.com/watch?v=UnFKh0WLVwo
Познавательное видео.
>>372696
#96 #372696
>>372695
хуйня для задротов
>>372698
#97 #372698
>>372696
Выпускник МГУ, подвизавшийся в Оксфорде (в одном из тамошних вузов), популярно поясняет, почему матфак сосёт.
>>372699>>372700
#98 #372699
>>372698
Блядь, я посмотрел, сложилось ощущение, что ему 15 рублей от мгу пришло.
#99 #372700
>>372698
Пиздец, посмотрел его видео, лютый зашкварник какой-то: "Учитесь на пятёрки, а то общество потом вас спросит о результатах", - и это, блять, математик? Где контркультурность, где сверхидеи, где демоническая одержимость? Это ёбанный карьерист - рак, убивающий математику, Гротендик и Серр дружно ссут ему в ебальник.
>>372701
#100 #372701
>>372700
Ну, по части оценок он как бэ прав. Если ты собираешься продолжать учиться за бугром, тебе лучше бы иметь пятёрки, так-то.
>>372702
#101 #372702
>>372701
Это бугру лучше бы не выёбываться, если он хочет, чтобы я у них продолжал учиться.
>>372703
#102 #372703
>>372702
Excellence programs всех сортов сейчас поссали на тебя
>>372706
#103 #372706
>>372703
Хуй знает что это за эксэлэнц програмс, поссал на тебя на всякий случай.
34 Кб, 604x463
#104 #372708
Шалом, перельманы.
Я в мтанах ваших ничего не знаю, но очень хочу разобраться. Например смысл среднего арифметического я понимаю, а смысл среднего геометрического нет. Нет, я знаю как оно считается блять, но суть нихуя не улавливаю при малых значениях СА и СГ очень близки друг к другу НО ПОЧЕМУ ОНИ НЕ ОДИНАКОВЫЕ Смысл матметодов тоже не понимаю, метод наименьших квадратов, например.
Кароч, посоветуйте книгу для совсем уж долбоебов тип меня, чтоб прям графически на графиках или может на словах в шутками-хуютками все доступно объяснялось.
#105 #372710
>>372708
Зачем тебе это?
13 Кб, 279x274
#106 #372714
>>372708

>смысл среднего геометрического


Среднее арифметическое в смысле показателя. Если у тебя есть e^a, e^b, e^c, среднее геометрическое будет e^{(a+b+c)/3}

>метод наименьших квадратов


Приближение данных линейным оператором. У тебя скажем есть вектора X_i из R^n, которым соответствуют вектора Y_j из R^m. Ты хочешь найти такой линейный оператор B:R^n->R^m, что B . X_i ~= Y_j (примерно равно). Самое простое -- минимизировать квадраты расстояний от B . X_i до Y_j, их сумму. Это и делает метод минимальных квадратов.
>>372716
#107 #372716
>>372714
B . X_i ~= Y_i
квадраты расстояний от B . X_i до Y_i
#108 #372717
>>372708
Корощ мне кажется тебе не нужна книга, достаточно гугла
>>372719
#109 #372719
>>372717
Можешь попробовать навернуть Босса "Интуиция и математика", хуй знает насколько рилейтед.
>>372720
#110 #372720
>>372719

>Босса


Его на dxdy обоссали.
#111 #372722
>>372720
Да мы тут сам дхду давно обоссали.
#112 #372724
>>372720
Пруф подгони, хочу поржать с идиотов. IRL Босс - это коллектив литературных негров, часть из которых очень квалифицированы.
>>372725
#114 #372732
>>372720
Там одни днища сидят
>>372734
#115 #372734
>>372732
Будто на сосаке одни успешные.
>>372735>>372743
#116 #372735
>>372734
А ты че их защищаешь, а? Ты вообще на чьей стороне, а? А!?
>>372736>>372737
#117 #372736
>>372735
С каких пор мне dxdy нельзя защищать?
>>372738
#118 #372737
>>372735
Какие ещё стороны? Ты упорот? Иди на бамбасе взорвись, если тебе деление на стороны нужно.
>>372738
72 Кб, 604x394
#119 #372738
>>372736
>>372737
Какие вы скучные
40 Кб, 1051x280
#120 #372739
Обычно как в шарагоинженерных предметах матана очень мало и он как-то легко заходил, а в тау как выяснилось все состоит из матана и такой даун как я не могу даже понять зачем тут нужен интеграл?
- Как я могу найти высоту воды в баке поделив, вроде как, площадь функции вытекания воды на площадь сечения.
- Почему площадь сечения оказалась в знаменателе?
>>372740>>372744
#121 #372740
>>372739

>матан

#122 #372743
>>372734
Поуспешнее будем)))
#123 #372744
>>372739
Интеграл дает тебе объем воды в баке. Объем разделить на площадь - выйдет высота.
#124 #372748
У меня накопилось МНОЖЕСТВО вопросов, аж два.

Почему теория множеств считается острием математики?
Я так почитал её и что то у меня сложилось впечетление что её изобрели специально для разжижения мозга математиков, чтобы они потом гомологии с гомотопиями на многообразиях анализировали и решали проблемы голода всего мира путем парадокса хаусдорфа-банаха-тарского.
Собственно, что то я разговорился. Что мне делать если мне надо математику выучить для того чтобы смочь в физику, в которой теория множеств не применяется в связи с её поразительной неэффективностью в ней, а все учебники по высшей математике начинаются со слов "возьмем множество натуральных чисел N..."? Не то чтобы мне было западло её учить, хоть и западло, но половина учебников, если не больше, построенны по принципу "долго и нудно вводим понятия чтобы на последней страничке написать "вот если ту хуйню вот туда поставить а потом вот сюда то что будет потом мы не знаем. Но мы работаем над этим""

Ну и второй вопрос, кто что может сказать про нестандартный анализ? Я почитал википедию, книги которые смог найти, понял его суть, понял что он легче теории пределов, но не понял почему его не преподают и не распространяют. Он же естественнее, или я его не понял вводим дополнительные числа, которые не подчиняются аксиоме архимеда и не представимы в виде суммы единиц, и с их помощью разрабатываем интегралы с дифференциалами
#125 #372766
>>372659

>пистех


Пистех (по легенде) создавался чтобы пилить атомную боньбу. От сюда получается что вершина математики на пизтехе - Уравнения Математической Физики, на довольно картофанном уровне. Плюс еще немножко разной дискреточки.
Никаких гротендико-фантазий там даже и близко не пробегало. Даже абстрактной алгебры в общем-то там толком не преподают хнык-хнык.
>>372771>>372978
#126 #372771
>>372766
Что значит "картофанный уровень"?
>>372997
220 Кб, 1024x768
1021 Кб, 2048x1365
155 Кб, 1280x800
31 Кб, 450x450
#127 #372772
>>372748

>Почему теория множеств считается острием математики?

>>372837
#128 #372774
>>372748
СВЯТАЯ ТОЛСТОТА
>>372837
#129 #372777
>>372748

> все учебники по высшей математике начинаются со слов "возьмем множество натуральных чисел N..."


И что главное: никак не определяют это множество, а если и определяют, то через какие-то трюки с рекурсией....
42 Кб, 604x330
#130 #372779
>>372777
Пусть дано множество M и отношение порядка < на нём...
>>372786
25 Кб, 511x604
#131 #372782
>>372777
vector<int> v;
#132 #372783

>Почему теория множеств считается острием математики?


Не считается.

>Я так почитал её и что то у меня сложилось впечетление что её изобрели специально для разжижения мозга математиков, чтобы они потом гомологии с гомотопиями на многообразиях анализировали и решали проблемы голода всего мира путем парадокса хаусдорфа-банаха-тарского.


Нет, её изобрели как попытку (более-менее удачную) подвести всю математику под единый язык и сделать арифметику и анализ одной наукой.

>Собственно, что то я разговорился. Что мне делать если мне надо математику выучить для того чтобы смочь в физику, в которой теория множеств не применяется в связи с её поразительной неэффективностью в ней, а все учебники по высшей математике начинаются со слов "возьмем множество натуральных чисел N..."?


Прочитать введение на 5 страниц в любом учебнике по анализу. Думаю, от получасового чтения не умрешь.

>Ну и второй вопрос, кто что может сказать про нестандартный анализ? Я почитал википедию, книги которые смог найти, понял его суть, понял что он легче теории пределов, но не понял почему его не преподают и не распространяют. Он же естественнее, или я его не понял вводим дополнительные числа, которые не подчиняются аксиоме архимеда и не представимы в виде суммы единиц, и с их помощью разрабатываем интегралы с дифференциалами


Милая идея, но бесполезная. Он использует сложные теоретико-модельные конструкции, для которых нужно очень хорошо понимать сущности некоторых метаматематических эффектов.

В рувики хуйня написана, кстати, определение производных через дуальные числа и нильпотенты высших порядков никакого отношения к нестандартному анализу не имеет, и это действительно довольно няшное определение, которое нужно если не брать за основное, то рассказывать о нём в упражнениях или замечаниях.
>>372837
#133 #372786
>>372779
ПО ИНДУКЦИИ
#134 #372837
>>372772
>>372774
А разве нет?

>>372777
Ну, в книжках это все на чистой интуиции определяется, мол это целые числа, это натуральные, это трансцендентные а это вообще все подряд в одной куче.

>>372783

>сделать арифметику и анализ одной наукой.


Зачем?

>Прочитать введение на 5 страниц в любом учебнике по анализу.


Как я писал выше, в связи с поразительной неэффективностью теории множеств вне математики это будет балласт тормозящий обучение.

>нужно очень хорошо понимать сущности некоторых метаматематических эффектов.


Это например каких?
>>372919
213 Кб, 1049x949
#135 #372870
Поясните за пикрелейтед, последний абзац. L^1 - пространство интегрируемых функций. Как складывать и умножать классы эквивалентности? Я так понимаю, как с классами вычетов по модулю: нужно выбрать по представителю, сложить/умножить представителей и это будет результат. По идее же еще нужно доказывать, что результат не зависит от выбора представителя?

В общем, поясните за общую ситуацию: мы разбиваем множество на классы эквивалентности и вводим на множестве классов эквивалентности какие-то операции; чтобы применять операции к классам, мы их применяем к представителям.
Какой должен быть порядок действий в этой ситуации?
>>372919
#136 #372913
Сколько-то тредов назад мне обещали обосновать почленное интегрирование ряда без равномерной сходимости. Ну так че?
>>372919>>373216
#137 #372919
>>372837

>Зачем?


Чтобы использовать результаты одного раздела в другом и вообще для душевного спокойствия и чувства единства.

>Как я писал выше, в связи с поразительной неэффективностью теории множеств вне математики это будет балласт тормозящий обучение.


Зато теория множеств поразительно эффективна в математике (что неудивительно, ведь именно за этим её создавали), поэтому если ты хочешь выучить именно математику, то тебе придётся выучить азы ТМ.

>Это например каких?


Существование нестандартных моделей и их неосязаемость в большинстве случаев.

>>372870
Нужно доказывать. Или доказать это один раз для всех нормированных векторных пространств. Правда произведение функций из L^1 вообще может не быть функцией из L^1, но это мелочи жизни.

>>372913
Я думал ты уже съебал давно. Напишу сегодня, лил.
>>372935>>373216
#138 #372935
>>372919
Нет, я хочу выучить именно физику. Но её не выучишь без математики. А математике сейчас учат только через теорию множеств.

Например? Что за модели и что за неосязаемость? Подробнее распиши.
>>372939
#139 #372939
>>372935
Потрать час на первую главу в Зориче. Ты бы давно уже выучил нужный тебе кусок ТМ, если бы не хныкал.

>Например? Что за модели и что за неосязаемость? Подробнее распиши.


Сложно расписать человеку не в теме. Если коротко, то мат. теория - это просто синтаксическая конструкция - множество формул (теорем) на некотором очень строго определённом языке. Модель - это когда мы значкам из нашего очень строгого языка сопоставляем элементы алгебраической системы так, что теоремы выполняются. У теории бывают довольно привычные модели, которые называются "стандартными" и довольно сюрреальные, которые называются "нестандартными".

Например интуиция о натуральных числах (sic) аксиоматизируется через аксиомы Пеано, аксиомы Пеано имеют стандартную модель: привычные нам натуральные числа {1,2,3,...} и имеют нестандартные модели, в которых есть нестандартные элементы, большие любого "стандартного" числа. Проблема в том, что по теореме Таненбаума я не могу построить рекурсивно аксиоматизируемую теорию такой вот нестандартной модели, поэтому говорить о них как бы можно - можно делать какие-то даже универсальные утверждение о их структуре (например, известен их порядковый тип), но нельзя "пощупать" их. Нельзя, например, в языке программирования сделать класс "нестандартное натуральное число" с как-то переопределёнными операторами умножения и сложения, так, чтобы все аксиомы выполнялись.
>>372960>>373063
#140 #372960
>>372939

>по теореме Таненбаума


Я уже который раз тебя вижу, анон. Сколько можно повторять? Он Tennenbaum, через е. https://en.wikipedia.org/wiki/Tennenbaum's_theorem
>>372964
#141 #372964
>>372960
Строго говоря, он Тонненбом - фамилия-то французская.
>>372966>>372971
#142 #372966
>>372964
В литературе он постоянно фигурирует как Тенненбаум, особенно в сочетании Соловей и Тенненбаум. Можешь проверить на матнете.
#143 #372971
>>372964

>французская


?????

танненбаум - ёлка
>>372972
#144 #372972
>>372971
А Тонненбом - математик.
>>372974>>372976
#145 #372974
>>372972
Не знаю такого. Знаю лишь американского Стенли Тенненбаума.
#146 #372976
>>372972
Фамилия немецкая, долбоеб.
#147 #372978
>>372666
Самый широкий математический кругозор ты получишь, учась на физика-теоретика не разлагающего в ряд Тейлора, а угорающего по калибровкам и по струнам в одном месте, и на математика в другом.
С последним достаточно ясно - НМУ и программа Вербицкого, а в первом проблема в том, что я знаю только одно место, где занимаются первым ФОПФ Пистеха, но это люто отнимает время, травмирует психику и давит на людей.
По не особо большому опыту общения - люди с ВШЭ и мехмата уровня конца бакалавра-начала магистратуры не особо знают за даже калибровки, в то время как пистехи некоторых кафедр типа ИТЭФа могут пояснить за Стандартную модель и КТП. Но за это знание придётся заплатить прохождением сотен говнокурсов и иным.
Правда, эти же пистехи могут знать расслоения только на уровне определения и расслоения Хопфа, чем дико выёбываться, но это уже другая история. Ещё есть проблема в том, что эти курсы придётся самому переводить с картофельного языка на нормальный, от чего будут гореть жопа и голова, да ещё и учить обезьянничество на уровне трёх теорем вместо одной нормальной теоремы Стокса. Либо страдать от отсутствия стипендии и нормального отношения к тебе, как это делаю я.

Прикладных математиков я бы тебе не советовал. Куча информатики и дискретного, логика без теории категорий, алгебраической геометрии вроде как тоже нет. Либо же есть, но так, что лучше бы и не было.

>>372766
Карасёв читал годовой курс, в котором пояснял за гомологии и характеристические классы векторных расслоений. По идее, он знает и К-теорию, и симплектическую геометрию, и нечто большее, но на пистехе таких людей единицы хотя разбирающихся знаю лично двух-трёх.
Ещё у ФИВТов на 3 курсе был курс про гомологии, но это где-то на базе и он не очень годный, как по мне.
>>372986>>372997
#148 #372979
Филологи в маттреде, спешите видеть!
>>372982
#149 #372982
>>372979
АТТИЯ
>>372983
#150 #372983
>>372982
ГЕЛЬФ А НД
>>372984
#151 #372984
>>372983
значение знаешь?
#152 #372986
>>372978
Дело вот в чем. Я люблю математику, но понимаю, что такой человек как я вряд ли когда-то станет ученым. К физике у меня отношение нормальное, готов ее учить, но... все же я ее понимаю несколько хуже, чем математику, и интереса к ней несколько меньше. Так что, для такого как я ФОПФ- не вариант. Но, может быт я чего-то не понимаю и ты меня переубедишь.
>>372991
#153 #372991
>>372986
Не идти на ФОПФ - выбор правильный (если только ты не являешься дико работоспособным или любишь скорости; к слову, в плане бессмысленной и просто нагрузки между факультетами не особо различие, поэтому дважды подумай, если всё же решишь выбрать физкек). Если МОЛОДОСТЬ ХАРДКОР не для тебя, ты не хочешь получить почётное звание местного сумасшедшего, а хочешь жить относительно тихо-мирно и изучать математику для себя - то матфак ВШЭ для тебя правильный выбор, мне кажется там, правда, общежитие далеко, но если есть возможность жить рядом - это не проблема. С третьего курса получишь возможность выбирать то, что нравится и не напрягает хоть экономика, что, кстати, даёт возможность безболезненно сменить специализацию в случае чего.
Об отсутствии невыносимой сложности говорит факт, что листочки с заданиями некоторые решают с репетиторами за не очень большие деньги, и что на матфак порой идут те, кто не поступил на нужный факультет.

Что касается необходимого материала - то >>372651-кун написал верно, за два курса ты получишь весь необходимый (или хотя бы 70%) материал для изучения им описанных дисциплин, далее - изучаешь их, а затем (в идеале - поняв 2-3 первых курса программы Вербита) - то, что желаешь, по книжкам.
И повторю совет - изучай физику (квантмех, теории поля, струны - только не задачки из ОБЩЕЙ ФИЗИКИ), дабы не отрываться в прямо таки совсем чистую математику. Вроде в ВШЭ даже есть курсы, правда, не знаю, насколько они хороши.
После этого, в принципе, должны появиться очертания всего происходящего в математике, а там уже поймёшь всё сам.
>>373006
#154 #372997
>>372978

>По идее, он знает и К-теорию, и симплектическую геометрию, и нечто большее


Ну охуеть теперь просто. Крепкого здоровья ему и творческих успехов.

>Правда, эти же пистехи могут знать расслоения только на уровне определения и расслоения Хопфа


Сомневаюсь что средний физтех сможет ответить что такое расслоение, да даже что такое многообразие. Только если несколько хардкорных ботанов с ФОПФа.

Мне как бы хотелось помечтать чтобы основная программа была что ли "поинтереснее", а то что там может даваться на спецкурсах - которые занимают от силы 5% нагрузки - ну не знаю хуй с ними

>>372771

>Что значит "картофанный уровень"?


Ну примерно так же как изучение интегралов на первых курсах сводится к задрачиванию формулок до изнеможения, так же УМФ на курсах постарше сводятся к задрачиванию стандартных подходов для стандартных задач.
Хотя сама эта тема безгранична и можно было бы рассматривать всякие современные физические теории, струны, топологические инварианты, работы Виттена и прочее.
Хотя видимо все это не нужно, а нужен очевидно только картофан.
30 Кб, 600x264
Комбинаторика #155 #372998
Привет матемач. Отправили к Вам.

Помогите решить задачку.
Дано, количество игральных костей - N, загаданное число - M. Все кубики 6-гранные.
Необходимо рассчитать вероятность выпадения загаданного числа.
>>373008>>374304
#156 #373006
>>372991

>изучай физику (квантмех, теории поля, струны - только не задачки из ОБЩЕЙ ФИЗИКИ), дабы не отрываться в прямо таки совсем чистую математику


Это ещё зачем?
#157 #373008
>>372998
Чёт хз. Эта задача решается как-то кроме полного перебора?
>>373027
#158 #373027
>>373008

Конечно решается. Нам же не надо знать все возможные расклады, нам нужна только сумма - соответственно мы можем свернуть результат суммированием.
#159 #373063
>>372939
С комплексными числами вполне себе оперируют и не заморачиваются поисками числа квадрат которого равен минус единице. А ведь такое число не "пощупать".

Я вот щас по мельком прочитал английскую вики про нестандартный анализ, там такое же написанно что и в русской по этой теме, разве что более подробно, с применением суперструктур и прочим подобным. ВРоде мне все понятно.
>>373375
#160 #373065
>>372655
Можешь понять, если у тебя ПОДХОДЯЩИЙ склад ума. Ведь все люди разные. Может ты сходу вкуришь и представишь себе двухмерную визуализацию трехмерной проекции пространства калби яу, а может будешь тупить над синусами с арктангенсами пять лет подряд и так и не поймешь их суть.
97 Кб, 605x474
#161 #373066
#162 #373099
Почему общую топологию не изучают в школах вместо того, что там изучают? Как минимум, для матана нужно знать про непрерывность, компактность, связность, метризуемость, полноту и т. д. То есть, без топологии даже интегральчики под картофанчик осознанно хуярить нельзя, потому что все доказательства опираются на топологию. Но ведь во всех вузах обучение построено так, что студенты вникают в доказательства. Разве школа не должна готовить к вузу?
#163 #373101
>>373099
Школьный реформатор, у тебя летнее обострение!?
>>373104
#164 #373103
>>373099

>Почему общую топологию не изучают в школах вместо того, что там изучают?


Непрактично. Её и в университетах часто не изучают.

>Как минимум, для матана нужно знать про непрерывность, компактность, связность, метризуемость, полноту и т. д.


Всё это делается для случая вещественной прямой.

>То есть, без топологии даже интегральчики под картофанчик осознанно хуярить нельзя, потому что все доказательства опираются на топологию.


В школе особо интегралов не берут.

>Но ведь во всех вузах обучение построено так, что студенты вникают в доказательства.


Доказательства мало кого волнуют, используются наглядные примеры — площадь под графиком и путь.

>Разве школа не должна готовить к вузу?


Школа воспитывает членов общества.
>>373104
#165 #373104
>>373103

> Школа воспитывает членов общества.


Справедливо.

>>373101
Я не он.
#166 #373107
>>373099
Давай рассмотрим на примере геометрии. Обычно ее преподают с 7 по 11 класс, 2 или 3 часа в неделю, получается около 180 часов в год, 900 часов за 5 лет.

При этом что она даёт ученику?
1) неформальные и порой неверные определения геометрических объектов
2) кучу забавных следствий из аксиом евклида, но без самих аксиом
3) умение записывать геометрические доказательства ужасным бюрократическим языком, неиспользуемым больше нигде

Где это пригодится в дальнейшем? Да нигде, это тупик.
>>373125>>373127
#167 #373125
>>373107
Геометрия учит мыслить. Верь в это.
#168 #373127
>>373107
Лол, так еще Вавилов говорил, что доказательства из школьной геометрии чуть ли не единственное полезное, что можно вынести из школы, т к эти доказательства являются доказательствами не только по меркам школьной, но и по меркам "взрослой" математики.
#169 #373129
>>373127
Почему?
>>373132
#170 #373132
>>373129
Не знаю. У Вавилова надо спросить.
312 Кб, 673x952
361 Кб, 673x967
411 Кб, 729x988
354 Кб, 653x969
#171 #373133
>>373127
С такой аксиоматикой и терминологией -- путь во взрослую математику открыт!
>>373142
#172 #373141
Так что, школьная геометрия говно и в вузе учат с нуля?
#173 #373142
>>373133
Конкретизируй претензии.
>>373157
#174 #373157
>>373142
Как это - через точки? Что такое: полуплоскость, наложение, неразвернутый угол, единица измерения отрезка, фигура?
Почему аксиомы представлены в виде каких-то бытовых утверждений о какой-то там плоскости в какой то там прямой? Зачем для конгруэнтности какие-то дополнительные аксиомы, неопределенное понятие фигуры?
>>373172
#175 #373158
>>373127
Вынести-то можно, только, учитывая то, что школьник изначально не знает, что важно (доказательство), а что второстепенно (формулки), а учителя не объясняют это, он и не выносит ничего из важного.
Это, конечно, верно, если человек учился в обычной школе, а не в математической или каком-нибудь приличном лицее.
Знаю как по своему опыту, так и по знакомству с многими людьми физ-мат специальностей.
Прекрасно помню, как мне уже в ВУЗе объясняли, что эквивалентность утверждений нужно доказывать как "Пусть выполнено А. Тогда В. Пусть выполнено В. Тогда А".
При этом, что характерно, незнание такого базового никак не повлияло на конечные знания математики.
#176 #373172
>>373157
Тебе непонятно, что такое наложение и полуплоскость? Тогда тебе не в школу, а в детский сад надо.
>>373195
#177 #373195
>>373172
Я так и знал, что что-то упускаю. Я же не ходил в детский сад, а именно там давали строгие понятия этих вот объектов. Гильберт видно тоже там не был, но в его времена вообще все неразвито было.
>>373221
#178 #373199
Как интеграл Лебега согласуется с "несобственными" интегралами"? Например, мы хотим вычислить интеграл от f(x) = 1/x^2 на [1; inf). Я думал ввести посл-ть функций f_n таких, что f_n(x) = f(x), если x <= n, и 0, если x > n. Тогда f_n сходится к f, но я не могу вычислить интеграл от f как предел от интегралов от f_n, потому что чтобы применить dominated convergence theorem, мне надо знать, что |f_n| можно ограничить интегрируемой функцией. Как быть?
#179 #373216
>>372919
>>372913
Ну че, давай уже. Я собираюсь читать про элементарные функции типа exp, sin и cos.
exp определяется через ряд. Насколько я понимаю, справедливость почленного дифференцирования этого ряда будет объясняться через равномерную сходимость или что-то типа того.

Что я уже знаю: меру Лебега на прямой, интеграл Лебега, monotone convergence theorem, dominated convergence theorem и как с помощью нее обосновать почленное интегрирование ряда (без равномерной сходимости).
Мне теперь надо разобраться с почленным дифференцированием и потом я буду читать главу про аналитические функции (это же так называется?)
>>373376
#180 #373221
>>373195
Определение не нужно делать строже, чем это требуется для работы. Провёл прямую, ткнул пальцем - вот и вот полуплоскости. Определение завершено.
#181 #373252
Какие разделы математики самые-самые новые среди известных местным анонам?
>>373254
#182 #373253
https://vk.com/wall-42366362_1024?reply=1118

>Российские учебники по математике, по которым мы учимся, это ТУПОЙ НАБОР ПСЕВДО-"УМНЫХ" СЛОВ. Который никакого отношения к математике не имеет.


>Я отработал преподавателем математики 12 лет и видел десятки самых ходовых учебников математики. Поэтому хочу рассказать, что в них не так.


>Приведу в пример популярный учебник Пискунова Н.С. "Дифференциальное и интегральное исчисления", 9-е издание. Вот, например, как Пискунов начинает раздел "Формула Тейлора":


>...


>1) Первая реакция на такое описание формулы Тейлора:


>ЧТО ЭТО ЗА ФИГНЯ? ПРИ ЧЕМ ЗДЕСЬ ВООБЩЕ ФОРМУЛА ТЕЙЛОРА.


>Даже определение формулы Тейлора у Пискунова написано лишь через три (!!!) страницы ниже.


>2) Автор пытается изо всех сил запутать читателя.


>Любой адекватный автор пишет текст по всем законам логики. Если уж надо писать про формулу Тейлора - так и надо писать: "Формула Тейлора это.... Она нужна для.... ".


>...


>5) Автор - не существует как личность.


>В учебнике Пискунова вы нигде не встретите слова "я занимался тем-то и тем-то...", "мой опыт показывает, что...", "я занимаюсь 30 лет применением математики в различных отраслях знаний".


>Это какой-то аноним. А ценность текстов анонима - нулевая. Наверно автор просто переписывал работы Ньютона, Эйлера и других великих, поэтому у него не было морального права написать "Я".


>А ведь основной принцип обучения - у кого учишься, тем и станешь. Учишься у сантехника - станешь сантехником. Учишься у Пискунова - станешь Пискуновым. А кто такой Пискунов? Почему он не пишет о себе в каждом разделе - как он применял этот раздел, как ему в жизни это помогло? Сколько он на этом заработал или сколько премий Филдса получил? Почему мы должны становиться Пискуновым, хотя не знаем, кто это, и как ему помогла его математика?


>...

59 Кб, 1068x368
#183 #373254
>>373252
Diagrammatic algebra и её подразделы. Появилась по популярной версии в 1987, по неофициальной аж в 1882.
http://mathoverflow.net/questions/168888/who-invented-diagrammatic-algebra

Суть такова.
https://arxiv.org/abs/1106.2128

Википедия про это мало что знает.
https://en.wikipedia.org/wiki/Category:Diagram_algebras
#184 #373256
>>373254
Благодарю. Ну же, анон, накидай еще.
#185 #373261
>>373254
Ого. Выглядит охуенно
#186 #373262
>>373253

> ценность текстов анонима - нулевая



Вот это сейчас обидно было.
#187 #373263
>>373254
Перескажите вкратце суть для Ъ.
#188 #373264
>>373253

>Учишься у Пискунова - станешь Пискуновым.


Единственная умная мысль в этом потоке бреда. Группа, к слову, просто феерическая, я впечатлился.
>>373373
#189 #373266
Реквестую годные книги по
- мат. анализу
- аналитической геометрии и линейной алгебре.

Заранее спасибо.
>>373269>>373270
#190 #373269
>>373266
Начинать с зорича, читать все книги параллельно.
>>373270
#192 #373296
Какой толк от этих картиночек? Можно ли с их помощью доказать тождество Бианки, или нет простите ебитесь с индексами?
#193 #373297
#194 #373303
>>373296
Передовые разделы математики хороши тем, что не нужно доказывать их полезность всем подряд. Если ты не в теме - иди своей дорогой, занимайся чем-то более консервативным, ты здесь не необходим, никто тебя сюда не зовёт, никто не пытается тебе что-то прорекламировать. Если тебе это не интересно - просто не занимайся этим, проходи по своим делам, отстань.
>>373317
#195 #373316
>>373296
Может и можно, зависит от того, какие картиночки.
#196 #373317
>>373303
Нинравица ухади!111 аргументы ни для миня!1111
>>373336
20 Кб, 480x360
#197 #373336
>>373317
Именно так. У нас тут наука. Вас никто не знал. Мы вас не знаем.
>>373338
#198 #373338
>>373336
Этот прекрасный аргумент во всю свою силу разворачивается, будучи применённым по отношению к рыбниковым и ведической математике.
>>373358
#199 #373358
>>373338
Древний русс в треде, все в всерод
#200 #373370
>>373253
Лол, человек после

> 18-летнего опыта применения математики


открыл для себя, что чистая математика не нужна инженерам. Ебать он умный.
#201 #373371
>>373253

> Современная профессиональная математика - очень простая и мощная, дает серьезные результаты и может служить источником сверхдоходов.



> Если вам нравится учебная математика (оторванные от жизни детские задачи с полным отсутствием логики) - тогда эта группа не для вас. Каждое сообщение в группе будет вас бесить и противоречить вашему мировоззрению))



> Также эта группа о том, как зарабатывать 10000 р/день с помощью математики. Как найти людей, кому нужна математика, какие математические услуги нужны клиентам, как математику работать с высокой производительностью, как продавать математические услуги.



> Эта группа о математике, которую поддерживает высшее научное руководство России - РАН и ВАК, об уровне, который принят в математических НИИ и научном сообществе.



Чет ору.
#202 #373372
>>373253

> Читать учебники по-математике, надеясь изучить математику - все равно что читать частушки Петросяна, надеясь изучить историю и политологию.



> Все что было в науке раньше 2010 года - ОФИЦИАЛЬНО НЕ СЧИТАЕТСЯ НАУКОЙ. А относится к "истории науки".



Чем-то напоминает местных анонов.
#203 #373373
>>373264

>Группа, к слову, просто феерическая


Вот что бывает без карательной психиатрии.
#204 #373375
>>373063
Для комплексных чисел вполне можно написать класс в языке программирования, который будет реализовывать комплексную арифметику (по крайней мере для Q(i) - точно можно).

>Я вот щас по мельком прочитал английскую вики про нестандартный анализ, там такое же написанно


Там ни слова о дуальных числах.

> ВРоде мне все понятно.


Это иллюзия.
#205 #373376
>>373216
Прости, инет отключили. Вообще dominated/monotone теоремы не являются полной заменой равномерной сходимости.

Поэтому я просто напишу эссе о "перестановках интеграла и производной".
>>373460
#206 #373392
Куда поступать, чтоб понимать о чем вы тут пиздите?
>>373393>>373400
#207 #373393
>>373392
Это эзотерическое знание которое можно постигнуть только при помощи духовных практик, а не петушинных вузов.
102 Кб, 800x600
#208 #373400
>>373392
Прочитать эти книги можно без всякого поступления.
>>373401>>373404
#209 #373401
>>373400
Ленг, Ван Дер Варден и Колг.Фомин говно. Дискач.
>>373412>>373415
#210 #373404
>>373400
по такой логике что угодно можно выучить без поступления + я ж сходу не пойму даже предисловие
>>373405>>373413
#211 #373405
>>373404

>по такой логике что угодно можно выучить без поступления


Так и есть.
>>373406
#212 #373406
>>373405
Сайенчую. Потом можно новую хронологию придумать или ещё какую-нибудь хуйню.
>>373407
#213 #373407
>>373406
А Фоменыч самоучка разве? Не знал.
>>373432
78 Кб, 584x444
#214 #373412
>>373401
Ты не выдвинул аргументов. Тебя можно опровергнуть, просто сказав слова "это не так".
#215 #373413
>>373404
Думаешь, что после поступления что-то волшебным образом изменится? Тебе всё так же нужно будет прочитать учебники, только вдобавок придётся большую часть дня тратить на лекции по физкультуре и ОБЖ.
>>373417>>373421
#216 #373415
>>373401
Ван Дер Варден устарел.
Ленг сух и перегружен.
Колмогоров Фомин устарел и сух и перегружен (где спектральная теорема? где азы C*-алгебр? где теория индекса? где котягории?)
>>373418
#217 #373417
>>373413
Сначала поступают в ебанутые вузы, а потом жалуются на ебанутый учебный план.
>>373420
#218 #373418
>>373415
Ван дер Варден не устарел.
Ленг минималистичен.
Колмогоров-Фомин - книжка для элементарного введения в анализ и для разъяснения, почему определения Шварца именно такие.
>>373424
#219 #373420
>>373417
Во всех вузах России есть принудительные физкультура, культурология, бжд, история, философия и русский язык, в части вузов есть обязательное православие.
>>373446
#220 #373421
>>373413
да, но там можно будет задать вопросы на темы, которые не понял, повшать чсв, решая задачи перед быдлогруппниками + будет мотивация учить, чтоб не вылететь
>>373423>>373426
#221 #373423
>>373421

>можно будет задать вопросы на темы, которые не понял


Только если ты не понял очередное эпсилон-дельта определение. На вещи посложнее преподы будут отвечать "я этого не знаю, это не нужно".
#222 #373424
>>373418
Ты в эти книги почти что не заглядывал, наверное.

>Ван дер Варден не устарел.


Алгебра без категорий? Их уже на первом курсе читают.

>Ленг минималистичен.


Ага, как и все Бурбаки, эталон минимализма ёба.

>Колмогоров-Фомин - книжка для элементарного введения в анализ и для разъяснения, почему определения Шварца именно такие.


А, ты тот говноед-бурбакист, тогда нахуй иди, ненавижу тебя.
>>373425
#223 #373425
>>373424
Категории есть у Ленга. Ван дер Варден уравновешивает минималистичность Ленга.

>тогда нахуй иди


Воспринимаю это как твою капитуляцию.
>>373427
#224 #373426
>>373421
Ты за 4 года в лучшем случае выучишь вещественный и комплексный анализ, и пройдёшь какой-нибудь спец.курс типа "современные методы в топологии", где дадут определение гомотопическим группам и всё. Курсы алгебраической геометрии и алгебраической топологии (считающиеся стандартом для математика де-факто) вообще не входят в обязательную программу. Алгебра и дифференциальная геометрия дико урезаны и читаются на уровне 19 века. Так что если будешь равняться на университет - максимум ботаном-заучкой ебучим, а не человеком, станешь. (Хотя в топ. ВУЗах интеллектуальный климат ничего так, обычно, что мотивирует.)
>>373444
#225 #373427
>>373425

>Воспринимаю это как твою капитуляцию.


Это "манямир" называется.
>>373428
#226 #373428
>>373427
Если ты взялся пояснять за книги, но вместо аргументов начал издавать ругань, то как тут ещё можно отреагировать?
>>373430
#227 #373430
>>373428
Нахуй пойти, например.
>>373431
23 Кб, 548x478
#228 #373431
#229 #373432
>>373407
В истории да. Ты же нормально примешь постулат о том, что историк-самоучка математики - днище ёбаное. То же и с математиком-самоучкой истории.
>>373434
#230 #373434
>>373432
У тебя предложение построено - пиздец: "... историк-самоучка математики ..." - бля. Понял, что ты хотел сказать, только после третьего прочтения.

Да аналогия не совсем корректная, мне кажется. Во-первых история не совсем наука, а во-вторых математика не совсем наука; но в другую сторону. По факту-то все успешные молодые учёные более менее ебашили сами, а университет воспринимали как баласт и формальную хуйню для построения научной карьере. На Тифаретнике так пишут, по крайней мере. Лил.
#231 #373444
>>373426
ну бля придется идти в погромизды, раз все равно математику в одного ботать придется, а так хоть какую то работу потом получу
>>373453
#232 #373446
>>373420
Во всех вузах РФ есть обязательные история и физкультура. Остальное наполнение зависит от вуза.
>>373448>>373465
#233 #373448
>>373446
Культурология тоже везде есть.
>>373462>>373491
#234 #373453
>>373444
У тебя времени не будет на то, чтобы на серьезном уровне заниматься и программированием и математикой. Если ты не ебучий гений, конечно.
>>373458
#235 #373458
>>373453
пиздец какой то, а как тогда всякие ученые это делают? Учатся до 30 лет, потом начинают работать?
>>373470
#236 #373460
>>373376
Давай. Хочется, например, чтобы описывалось, почему ряд exp(z) можно почленно продифференцировать и за счет этого
[exp(z)]' = exp(z).

Все-таки, получается, что равномерная сходимость нужна и ты был неправ, когда сказал, что это лишняя глава в Зориче.
>>373470
#237 #373462
>>373448
Нет, не везде.
#238 #373465
>>373446
Во ВШЭ на 1 курсе всем, кто учился в корпусе на Кирпичной, ставили 10 автоматом по физкультуре, потому что зала для занятий не было. Такие дела. Истории тоже не было.
>>373466>>373475
#239 #373466
>>373465
Забыл сказать: объясняется это тем, что любой вуз, имеющий статус НИУ, может сам себе составлять программу. Но пидорашьи вузы предпочитают оставить историю и физкультуру даже если имеют этот статус.
>>373493
#240 #373470
>>373458
Да. Изредка им выдают гранты и стипендии. Более того, всё, что тебя ждёт в идеале (даже если ты возьмёшь PhD и получишь postdoc-позицию в престижном месте) - это 30-50к$ в год, что смехотворная зарплата по сравнению с аналогами в ИТ-сфере.
Как по мне путь учёного - это сорт оф путь монаха, и он включает в себя некоторый отказ от социальных благ.

>>373460

>Все-таки, получается, что равномерная сходимость нужна и ты был неправ, когда сказал, что это лишняя глава в Зориче.


Окей, я чуть ослаблю утверждение. Она нужна, но не так, а в контексте функ. анализа и всяких там модов сходимости.
>>373473
#241 #373473
>>373470

> 30-50к$ в год


> сорт оф путь монаха


Нихуя ты зажрался.
>>373476
#242 #373475
>>373465
Раз вам ставили автомат по физкультуре, значит она у вас все-таки была, и исключить ее из программы не могли, несмотря на отсутствие материальной возможности проводить занятия. Ок?

Мне, кстати, лекции по истории и культурологии запомнились больше всего. Надо было все-таи на лингвиста поступать.
#243 #373476
>>373473
Это ниже средней в СШПшках https://en.wikipedia.org/wiki/Household_income_in_the_United_States а конкуренция у тебя будет просто дикая.
>>373477>>373623
#244 #373477
>>373476
Так у среднего пиндоса свое кондо и 3 машины. Зачем нормальному человеку это надо?
>>373479
#245 #373479
>>373477
Тебе просто на то, чтобы есть и жить не в гетто будет уходить уже овер дохуя. Цены на одежду там тоже не самые маленькие. Я просто говорю, что в цивилизованном мире квалифицированный PhD прошедший дико жёсткий конкурс на работу, требующую огромного творческого потенциала, знаний и интеллекта получает в 3 раза меньше, чем стажёр-студент в Майкрасофте, который просто хорошо знает ЯП и более-менее эрудирован.
>>373534
#246 #373491
>>373448
Нет.
#247 #373493
>>373466
Они в какой-то степени правы, конечно, ведь от спец. предметов нужно как-то отвлекаться. Только делать это надо не так топорно. Например, можно давать на выбор много курсов всякого барахла, да ту же культурологию, может найдутся любители, а не делать это обязаловкой.
>>373497>>373639
#248 #373497
>>373493
Ну делают они это крайне нестарательно. Например, вводят небольшой курс философии, не специализируя его, в результате вместо интересного рассказа про кризисы математики и науку в рамках философских школ, читается обычный невнятный курс истории философии "одна лекция 5 философов". А потом еще и сдавать что-то требуют из этого нерелевантного говна. Ну а про преподавание устройства ПК и работы в TURBO pascal в 2016 году и говорить нечего.
#249 #373517
Задача, которую никто так и не решил:
Найти количество (угадайте ответ хотя бы!) всех неизоморфных плотных порядков без концов, у которых каждый собственный начальный отрезок счётен.
>>373533
#250 #373533
>>373660
#251 #373534
>>373479

> Цены на одежду там тоже не самые маленькие.


Хожу в спортивках с рынка.
https://youtu.be/L24GfNGMf9w?t=754
#252 #373608
Если морфизм пучков эпи на слоях, то он эпи как морфизм пучков?
>>373659
#253 #373616
Я тут подумал, что когда мы говорим "возьмем любую функцию из R в R", мы используем аксиому выбора. То есть без аксиомы выбора мы так не можем сделать?
>>373617>>373618
#254 #373617
>>373616
Нет, мы здесь используем правило вывода из логики предикатов: введение квантора всеобщности.
>>373625
35 Кб, 497x513
#255 #373618
>>373616

>Я тут подумал, что когда мы говорим "возьмем любую функцию из R в R", мы используем аксиому выбора. То есть без аксиомы выбора мы так не можем сделать?


По-моему это шиза.
>>373625
#256 #373623
>>373476

>Это ниже средней в СШПшках


Интересно, а можно где-нибудь найти динамику? Когда всё по пизде пошло?
#257 #373625
>>373618
>>373617
Ну типа для каждого x из R надо выбрать значение f(x).
#258 #373639
>>373493

>Они в какой-то степени правы диды говно жрали неча и нам начинать хорошо жить


Ой блядь дибил. Хорошо хоть пока в магистратуру и пхд никто не не додумывается это говно пихать.
>>373642
#259 #373642
>>373639
Ничего, что во всех так любимых тобой западных университетах тоже есть пресловутые гуманитарные курсы? При чем тут деды, дурашка?
>>373650>>373651
#260 #373650
>>373642
Неси пруф на то, что в западных университетах есть принудительные физкультура, бжд и культурология, которые нужно сдавать и из-за которых можно вылететь.
>>373652
#261 #373651
>>373642
Я про запад ничего не говорил, это ты уже сам додумал.
>>373655
#262 #373652
>>373650

> гуманитарные курсы


> физкультура



Алсо, гуглом пользоваться научись.
>>373654
#263 #373654
>>373652
Читай дискуссию глубже одного поста. Ну и пруфы неси.
#264 #373655
>>373651
Ну так и про дидов ты тоже сам додумал. Справедливый размен, как мне кажется.
#265 #373659
>>373608
бля проебался.
короче вопрос такой: правда ли, что если морфизм пучков эпи на сечениях, то он эпи как морфизм пучков?
>>373667>>374226
#266 #373660
>>373533
Нет.
#267 #373667
>>373659
Что за пучок и нахуй он нужен?
>>373780>>373805
#268 #373780
>>373667
лол картофан чтоле?
>>373788
#269 #373788
>>373780
Иди в пизду понял меня?
#270 #373805
>>373667
На энвики написано хорошо очень.
#271 #373845
>>372131 (OP)
Каковы взгляды на логику у Гильберта, Цермелло, Пуанкаре, Шрёдера, чем различаются?
>>373850
#272 #373850
>>373845
Блядь, у нас на факультете за такие вопросы убивают нахуй
#273 #373856
>>373857>>373871
#274 #373857
>>373856

>антоним гомеоморфизма


быстрофикс
#275 #373871
>>373856
Если такой термин и есть, то не общепринятый.
>>373874
#276 #373874
>>373871
может есть синонимы?
я надеялся найти информацию о чём-то типа отображения-выворачивания не обязательно непрерывного
>>373879
#277 #373879
>>373874
Прочитай про выворачивание сферы и про то, как процесс её выворачивания формализуется. Единственное, что пришло в голову.
>>373888
1661 Кб, 170x170
#278 #373888
>>373879
спасибо
тор, оказывается, тоже можно вывернуть
>>373889
#279 #373889
>>373888
Тор тоже можно, но твоя пикча никакого отношения к этому не имеет, так как на ней выворачивается проколотый тор, а не тор.

https://www.youtube.com/watch?v=kQcy5DvpvlM
#280 #373982
Кто-нибудь знает этимологию слов "мономорфизм" и "эпиморфизм"? Нашел сайт со значениями греческих приставок:
https://www.learnthat.org/pages/view/roots.html
Если учесть, что эпиморфизм - это аналог сюръективной функции, то понятно, почему он так называется. А почему мономорфизм так называется? Потому что прообраз одноэлементного множества относительно инъективной функции состоит из одного элемента?
#281 #374226
>>373659
очевидно да
80 Кб, 549x207
#282 #374284
Есть какие-то видео на ютубе, которые показывают гомеоморфизм пикрелейтед?
>>374288>>374403
#283 #374287
>>372691
учусь там
конкретно сказать сложно, но они вроде бы пытаются, что то делать в положительном направлении, программу потихоньку меняют.
#284 #374288
>>374284
Каким образом ты хочешь гомеоморфизм на видео показать?
>>374290
#285 #374290
>>374307>>374312
#286 #374304
>>372998
если число м больше н но меньше 6н то 1/6*н
#287 #374307
>>374290
при наглядном изображении гомеоморфизма тут просто петля пересечет сама себя и получится тор как тор
#288 #374312
>>374290
Интересно очень. А гомеоморфизм между чем и чем там показан?
#289 #374321
Зачем нужна обычная связность? В каких случаях недостаточно линейной связности?
>>374333>>374335
#290 #374333
>>374321
Линейная связность очень "гомотопическая" конструкция, её определение завязано на R, в то время как связность - общетопологическая и её определение ни на каких отдельных пространствах не завязано. Поэтому связность удобнее доказывать там, где рассуждения "общетопологические", а не "гомотопические". Линейную связность - наоборот.
#291 #374335
>>374321
во всяких динсистемах или еще каких-то диффурах естественно появляются связные не линейно связные пространства
кажется
#292 #374403
>>374284
Базарю, что ты хочешь не гомеоморфизм, а гомотопию. Это не одно и то же.
#293 #374417
Кстати о топологии. Сколько дыр имеет K-дыров? Сколько ручек?
>>374475>>374478
#294 #374475
>>374417

> Сколько дыр имеет K-дыров?


K, очевидно.
#295 #374478
>>374417

>K-дыров


Это который размазан?
>>374480>>374490
#296 #374480
>>374478
Р-мзн.
#297 #374490
>>374478
Нет, который шахмат К-дыров.
3978 Кб, 500x500
#298 #374499
Посоветуйте учебник по теории типов энтри левела.
>>374500
#299 #374500
>>374503
#300 #374503
>>374500
Это достаточно фундаментальная книжка?
Мне для TCS
>>374504
#301 #374504
>>374503
Если фундаментально в том, что большинство быдлокодеров ее бросают на 15 странице, то да.
>>374505>>374519
#302 #374505
>>374504
Большинство быдлокодеров и матан вузовский не осиливает. Мне интересно, это книга уровня Винберга/Городенцева по "Алгебере" в своей области?
#303 #374519
>>374504
Ты сам за сколько её прочитал?
115 Кб, 585x578
#304 #374549
Вопрос:

Можно ли представить единичный элемент в группе подстановок в виде произведения нечетного количества транспозиций, и почему?

И еще, что это на пике?
>>374554>>374555
#305 #374551
Извиняюсь за нерелейтед, но это правда, что после 25 мозг перестаёт развиваться? Просто недавно услышал такое заявление, что после 25 начинать учить математику, все равно что головой об стену долбится.
>>374553
#306 #374553
>>374551
Смотря какие у тебя цели. Профессиональным математиком ты уже не станешь, а для удовольствия можно хоть в 100 лет.
84 Кб, 301x453
#307 #374554
>>374549

>Можно ли представить единичный элемент в группе подстановок в виде произведения нечетного количества транспозиций, и почему?


Нет, знак перестановок.

>И еще, что это на пике?


Обычное бесконечное бинарное дерево. Как его развязать -- одновременно на всех уровнях сделать то, что на пикрелейтеде.
>>374557>>374567
#308 #374555
>>374549
Это конструкция сферы Александрова.
>>374557>>374559
#309 #374556
И оно не развязывается.
#310 #374557
>>374555
>>374556
Сцук( А я надеялся, что можно провернуть >>374554 одновременно
#311 #374558
Сфера Александера*
>>374559
#312 #374559
>>374555
>>374556
>>374558
В википедии написано, что эта конструкция называется еще дикая сфера. И определение "дикости".
Вложение стандартной двумерной сферы в евклидово трехмерное пространство называется диким тогда и только тогда, когда оно не продолжается до вложения окрестности этой сферы в евклидово пространство.

Почему оно не продолжается до вложения окрестности? возьмем просто открытый шар из нашего пространства, содержащий в себе данную конструкцию. вот и вложение, продолжающееся до окрестности.

Где ошибка?
>>374587
#313 #374561
Анон, как можно доказать делимость натуральных чисел на пять, заканчивающихся 0 и 5 соответственно?

Являются ли очевидные примеры доказательством

>10:5=2


>25:5 = 5


не думаю, поэтому и прикатил сюда ?
>>374564
#314 #374564
>>374561
Подойдёт ли подобная форма как доказательство?

* -любое целое число
если к любому целому числу приписать 0 или 5 в конце - считай, оно делится на 5.

Всё тот же >>374561 - кун
#315 #374567
>>374556
Чувак, чё-то я нигде не нашёл пруфы, что оно не развязывается, как и ошибки в способе >>374554

Давай-ка ты завезёшь их.
>>374587
#316 #374568

>приписать 0 или 5 в конце - считай, оно делится на 5.


Это злоупотребление русским языком.

Вот так четче будет:
Пусть а - нат. число, заканчивающееся на 0,
тогда его можно записать как а=10k => a делится на 5, т.к 10 делится на 5.(свойства делимости).

аналогично, если заканчивается на 5, то a=10
k+5 => делится на 5.
>>374570
#317 #374570
>>374568
Можно ли сказать, что натуральное число - дискретная величина т.к. является исчисляемой?
>>374574>>374604
204 Кб, 1032x774
#318 #374573
как четко доказать, что любая группа допускает иньективный гомоморфизм в группу подстановок(не обязательно конечного множества)?

Можно попытаться явно задать этот гомоморфизм f.
Пусть f(1 - в первой группе)=1 - единица в группе подстановок.
А дальше как, что бы почетче?
176 Кб, 720x951
#319 #374574
>>374575
#320 #374575
>>374574
Что, вопросы нюфань здесь не котируются?
>>374576
#321 #374576
>>374575
Ну скажи, что ты понимаешь под исчисляемостью
>>374579>>374581
#322 #374579
>>374576
То, что можно вычислить.
*вычисляемость, обосрался малость
#323 #374581
>>374576
то, что можно перечислить
116 Кб, 1666x1784
#324 #374583
А как же рекурсивные функции?
#325 #374584
>>372131 (OP)

>самой малой эпсилон окрестности этой границы.


Ясно.
#326 #374585
>>372366

>сигма-алгебра


Что?Что это?
>>374586>>374621
#327 #374586
>>374585
Какая-то Йоба с операцией счетного объединения множеств.
#328 #374587
>>374559

>вот и вложение, продолжающееся до окрестности.


Нет. Ты неправильно понимаешь слово "вложение". Вложение - это отображение из обычной сферы в сферу Александера. Нужно продолжить его до отображения eps-окрестности сферы в eps-окрестность сферы Александера.
>>374567
Ты очень странно искал
https://en.wikipedia.org/wiki/Alexander_horned_sphere
The exterior is not simply connected, unlike the exterior of the usual round sphere;
>>374600>>374616
2013 Кб, 500x500
#329 #374598
Верно ли, что любая группа из шести элементов изоморфна или группе перестановок S3, или произведению двух нетривиальных групп? И почему?
>>374661>>374685
#330 #374600
>>374587
а, спасибо
1953 Кб, 500x375
#331 #374602
Может в телеграме конфочку запилим?
>>374682
#332 #374604
>>374570
У тебя каша в голове.
#333 #374613
Не кормите школьника, для них есть отдельный тред.
>>374628
#334 #374616
>>374587

>The exterior is not simply connected, unlike the exterior of the usual round sphere;


Лол, это по-твоему означает неразвязываемость? ахахахахахахахахахаа, лол, ну ты тупой ппц.

У тора внешность тоже не односвязная, да и у бесконечного бинарного дерева, которому по моему рассуждению равносильна эта хуйня.
>>374674
#335 #374621
>>374585

>Семейство подмножеств множества X называется сигма-алгеброй над X, если объединение его счётного подсемейства является его элементом, пересечение его счётного подсемейства является его элементом, дополнение его элемента является его элементом.



Написано же.
#336 #374628
>>374613
Какого из?
#337 #374661
>>374662>>374685
#338 #374662
>>374661
Школьник, иди отсюда, тебе двумя тредами ниже.
>>374670
#339 #374670
>>374662
Хуй соси, даун.
#340 #374674
>>374616
Пошёл нахуй, мразь.
#341 #374682
>>374602
Да ну. Нахуя?
>>374686
#342 #374685
>>374661
как это относится к решению этой >>374598
задачи?
>>374696
#343 #374686
>>374682
чятик, в котором всегда кто-то есть
#344 #374687
Верно ли, что любые два векторных пространства с базисом Гамеля мощности A изоморфны? Два пространства со счётным базисом всегда изоморфны, да?
>>374688>>374718
#345 #374688
>>374690
#346 #374690
>>374688
А как доказать?
>>374691
#347 #374691
>>374690
Биективно отобразить базис в базис и проверить, что отображение будет линейным (по определению базиса).
#348 #374693
Почему предмет в вузе называется "дискретная математика"? Я правильно понимаю, что нет такого раздела математики?
>>374694>>374698
#349 #374694
>>374693
Раздел на arxiv.org есть, научные журналы есть, люди, идентифицирующие себя специалистами в дискретной математики есть - значит раздел есть.
#350 #374696
>>374685
Это группа из шести элементов, которая ни S3 ни произведение нетривиальных.
>>374697
#351 #374697
>>374696
Z/>>>>>>->>5<<-<<<<<<<Z группа из 6 элементов?
>>374699
#352 #374698
>>374693
Дисциплины в вузе и не обязаны быть "научными областями".
#353 #374699
>>374697
Ну Z/6Z.
>>374700
#354 #374700
>>374699
Это которая Z/2Z x Z/3Z?
>>374704
#355 #374704
>>374700
Ну да.
#356 #374718
>>374687
Нет, неверно. Бывают два неизоморфных пространства с базисом мощности алеф-нуль.
>>374746>>374775
#357 #374746
>>374718
Хуйню несёшь, либо троллить (ололо над разными полями гыгы))))
>>374765
#358 #374750
Ну и зачем?
#359 #374754
Что-то мне наивная теория множеств напоминает тригонометрию в школе. Есть набор формул, который я задрочил до полного автоматизма, пока занимался общей топологией и теорией меры, и теперь их использую чисто механически, не задумываясь.
>>374763
#360 #374763
>>374754
По каким книгам занимался общей топологией и теорией меры?
>>374793
#361 #374765
>>374746
Конечномерные пространства однозначно характеризуются размерностью. Бесконечномерные - нет.
>>374773
#362 #374773
>>374765
Тебе мамка так сказала?
А ниче, что биекция базисов продолжается до изоморфизма векторных пространств?
>>374778
#363 #374775
>>374718
Пример в студию.
>>374797
#364 #374778
>>374773
Валяй, попробуй доказать. А я посмеюсь.
>>374796
#365 #374793
>>374763
Топологией по разным. Теорией меры по Folland, Real Analysis: Modern Techniques and Their Applications.
>>374801
#367 #374797
>>374775
А я всё ещё жду пример. Спойлер: не дождусь
#368 #374801
>>374793
Как тебе книга Миши?
>>374809
#369 #374809
>>374801
Не решал, посмотрел только. Те топологические концепции, которые в матане используются, там есть, а про фундаментальную группу, накрывающие пространства и прочее ничего не могу сказать, потому что я картофан.
>>374813
#370 #374813
>>374809
Ну, то есть, я ориентируюсь в материале первых 7 листков, а в последних 3 ничего не знаю. Первые 7 листков норм вроде. Лекции не смотрел. Стоит с опасением к Вербиту относиться и не решать все подряд, потому что он любит пихать всякое говно типа p-адических чисел.
>>374815
#371 #374815
>>374813
почему это р-адические числа это говно?
>>374821>>374824
#372 #374821
>>374815
А зачем они в матане? В общей топологии?
#373 #374824
>>374815
Ну объясни, зачем они нужны человеку, который не занимается профессионально алгеброй или теорией чисел.
>>374831
#374 #374831
>>374824

> профессионально теорией чисел


Фикс.
82 Кб, 1313x385
125 Кб, 1309x837
115 Кб, 1249x670
#375 #374870
Рекомендовавший Зорича анон (или кто угодно, кто в теме), поясни за погрешности, пожалуйста. Как происходит переход, обведённый синеньким? Почему первая дробь по модулю меньше второй?
#376 #374898
>>374870
деление на число меньше 1, это по сути умножение на обратное, поэтому и больше
>>374911
#377 #374911
>>374898
Но ведь эти две дроби не обратны друг другу.
>>374930
#378 #374929
>>374870
Серьёзно, аноны, помогите. Что за магия тут использована?
#379 #374930
>>374911
Дробь делят на единицу плюс что-то, допустим на 1.2, дробь делят на единицу минус что-то допустим на 0.75. Какое частное больше?
>>374931
#380 #374931
>>374930
Но ведь бета, делённая на игрек с чертой, не обязана быть положительным числом.
#381 #374932
>>374870
Seichas posmotru.
#382 #374934
Для любого u т.ч. |u|<1 выполнено неравенство

|1/(1+u)| < 1/(1-|u|)
#383 #374935
>>374934
Почему?
>>374937
#384 #374937
>>374935
Неравенство с модулями 7-8 класс.

Давай разбираать случаи
0<=u<1
1/(1+u) <= 1/(1-u)
1+u > 1-u
2u >=0 верно

-1<u<=0
1/(1+u)<=1/(1+u)
верно
>>374941
#385 #374938
1+u >= 1-u
точнее
#386 #374939
>>374934
Пусть u = -2.
|1/(-1)| = 1
1/(1-2) = -1

1<-1 - ложь.
>>374940
#387 #374940
>>374939

>u т.ч. |u|<1


Кредо олимпиадника - 30% решения задачи - это правильно прочитать условие.
#388 #374941
>>374937
Окей. Спасибо. В моей школе не было седьмого класса просто.
#389 #374944
>>374934
Если что, при u=-1/2 там будет равенство.
>>374945
#390 #374945
>>374944
и при u=0 тоже, да, <= нужно, конечно же.
71 Кб, 1334x752
25 Кб, 1273x262
#391 #374976
B(X) - пространство ограниченных функций из X в R, наделенное sup-нормой. Поясните за последнюю строчку на пикрелейтед 1. Тут же наебка какая-то, да? На основании чего мы сделали вывод, что fm(x) под модулем переходит в f(x) при m -> Inf?
>>374984
#392 #374977
В чём суть capset problem? Нахуя это нужно?
#393 #374984
>>374976
А ты подумай. Любая посл-ть Коши сходится к своему пределу. У тебя там равномерная сходимость (независимо от точки).
#394 #375037
Как теорию категорий учить? Есть ли учебники, которые, например, объясняют её в связке с другими разделами?
#395 #375050
>>375037
Маклейн
а
к
л
е
й
н
#396 #375119
>>375037
Аводи лучше.
#397 #375174
>>375037
Современная - только в статьях. Старая - кто угодно, Маклейн - канон.
#398 #375186
Как вывели теорему ньютона лейбница? Ну вот с дифференцировнием все понятно, приращения, сверхреальные числа, стремление предела к нулю. А как заметили что дифференциал интеграла равен внутриинтегральной функции? Это сейчас все ОЧЕВИДНО, ЧТО И ТРЕБОВАЛОСЬ ДОКАЗАТЬ, а как ньютон с лейбницем заметили это?
Как они интеграл прикрутили к математике через дифференциал? Сидели вычисляли их, а потом лейбниц вскочил с криком "НУ ЕБАТЬ, СМАРИ КАКАЯ ХУЙНЯ, НЬЮТОША!!", ньютон глянул и сказал "Мммм, прикольно" и пошли вдвоем всем рассказывать что они заметили?
#399 #375190
>>375186
На вики же аж целых два параграфа на эту тему: geometric meaning и physical intuition.
>>375208>>375212
#400 #375200
>>375186
Короч пили они вдвоём, наутро протрезвели, бац, смотрят - блин, формула! Кто из них открыл? Зачем открыл?.. Чтоб долго не разбираться, назвали формулой Ньютона-Лейбница.
#401 #375207
>>375186
Рассмтрим простейший случай. Пусть точка движется с неотрицательной скоростью v(t) в течение промежутка времени [a, b]. Пусть путь, пройденный точкой на момент времени x, равен S(x). Ясно, что если v(t) - ступенчатая функция, то надо сложить площади прямоугольников под графиком скорости, чтобы получить весь путь. Но предположим, что v не ступенчатая. Тогда будем приближать площадь под графиком "бесконечно малыми" прямоугольниками. Если считать, что при "бесконечно малом" приращении времени dt скорость не меняется, то приращение пути dS можно считать равным площади прямоугольника dt * v(t). Весь пройденный путь равен сумме таких "бесконечно малых" прямоугольников.

Такие рассуждения, конечно, являются нестрогими, особенно слова "бесконечно малые". Но я думаю, что именно как-то так и догадались.
10 Кб, 833x254
#402 #375208
>>375190
И чем там за слово то?
>>375213>>375217
#403 #375209
>>375186
Производная это скорость интеграл это путь (функции от времени).
>>375211>>375215
#404 #375211
>>375209
Но ведь нет никаких оснований проецировать это с скорости и пути на все остальные функции.
>>375214
#405 #375212
>>375190
Так самые примитивные конструкции интеграла (по Риману и по Коши) никак и не связаны с дифференциалом, только на интуитивно-интенсивном уровне.
>>375213>>375216
#406 #375213
#407 #375214
>>375211
Если F' = f, то f ведет себя как скорость, потому что определение производной такое.
#408 #375215
>>375209
Я имею виду, вот допустим человечество ВСЕ, пленшеты сели, двачи недоступны, и чтобы подняться хотя бы на уровень 19 века нужна математика. Вот я лично её знаю более менее, а как мне объяснить будущим постчеловекам почему дифференциал интригала равен внутринтригальной функции? Сказать "Уверуй, ибо ваистину" не пойдет. Он скажет "Ниверю" и будет по своему прав,итак как я не могу доказать что надо верить в это.
>>375219>>375222
#409 #375216
>>375212
Это какие такие, примитивные?
>>375219
#410 #375217
>>375208
infinitesimal конечно же.
>>375218
#411 #375218
>>375217
Почему оно в спаме? Мод не понял суть гиперреальных чисел?
>>375224
#412 #375219
>>375215
Что тебе непонятно в объяснении на вики? Нарисуй криволинейную трапецию от x до x+h и покажи, что её высота равна примерно f(x) если h очень маленькое. Отсюда следует Int[x..x+h] f(x) dx ~ h * f(x)
>>375216
По Риману и по Коши, я ведь в скобках написал. Значок "dx" в интеграле по Риману - это не дифференциал, а просто часть синтаксиса.
>>375221
#413 #375221
>>375219
Ну вот возьмем квадрат функцию. Расчертим её до двух. Площадь под двумя равна 7/3. Добавим к двум 0.1. ПО твоей площади выходит что площадь примерно равна 0.7/3=7/30. Равзе это так?
>>375223
#414 #375222
>>375215

> вот допустим человечество ВСЕ, пленшеты сели, двачи недоступны, и чтобы подняться хотя бы на уровень 19 века нужна математика


> как мне объяснить будущим постчеловекам почему дифференциал интригала равен внутринтригальной функции?


Пиздец ты еблан. Тебе еще стоит на курсы выживальщиков в дикой природе записаться и начать готовиться к зомби-апокалипсису, строить подземный бункер у себя на даче.
>>375226>>375264
#415 #375223
>>375221
Площадь между 2 и 2.1 под графиком x^2 примерно равна 0.1 (2)^2 = 0.4
и по мне так оценка неплохая: https://www.wolframalpha.com/input/?i=integral+x^2+from+2+to+2.1
>>375226
#416 #375224
>>375218
Была ситуация.
#417 #375226
>>375222
Какая мощная аргументация.

>>375223
Ну да, подходит примерно.
Но я все еще не вижу связи с интегралом. У квадрата же он равен куб делить на три.
>>375227
#418 #375227
>>375226
Ещё раз попытайся объяснить что тебе непонятно. Тебе нужны максимально интуитивные и простые аргументы к тому, что производная у функции F(x)=int[0..x] f(t) dt равна f(x)?
На вики их аж целых два - в соответствующих двух абзацах (один из них я даже тут кратко пересказать умудрился). Нужно строгое доказательство? В любом учебнике анализа. Или что-то ещё?
>>375230
#419 #375230
>>375227
Интеграл это сумма значений функции на сверхмалых промежутках умноженных на длинну этих самых промежутков, так? Так.
Где связь с производной блять? Длинны промежутков через производную вычисляются? Значения функции через производную?
>>375232
#420 #375232
>>375230
Ну вот если брать интеграл по очень маленькому промежутку, то он (в силу непрерывности функции) будет примерно равен длине этого промежутка на значение в какой-то точке этого промежутка. То есть F(x+h) - F(x) ~ h*f(x). А теперь подели это выражение на h и устреми h к 0.
>>375236
#421 #375236
>>375232

F(x+h) - F(x)~h F(x)=F(x+h)/h - F(x)/h ~ h f(x)/h= (F(x+h)-F(x))/h ~f(x)

Так что ли?
>>375237
#422 #375237
>>375236
Я хотел, чтобы ты увидел, что F'(x) ~ f(x)
>>375240
#423 #375240
>>375237
Так, подожди, я же только что производную сделал по сути. Так, и что, получается интеграл это куча околонулевых производных? Н ведь это же ряд по моему я в ряды еще не сильно углублялся
>>375245
#424 #375245
>>375240

>Так, и что, получается интеграл это куча околонулевых производных?


Нет. Интеграл - это просуммированная куча значений функций помноженных на маленькие промежутки.

>Н ведь это же ряд по моему я в ряды еще не сильно углублялся


Интеграл концепция чуть сложнее, чем ряды, если её формализовывать. Так что если хочешь какой-то строгости - то придётся изучать всё последовательно, а если хочешь интуиции - то тут всё предельно просто и описано на той же вики.
>>375250
#425 #375250
>>375245
Все равно не вижу связи.
Ты меня никак не можешь ткнуть в суть, которая наверняка проста до безобразия, но которую я в упор не вижу.
Еще раз.
Интеграл это сумма значений функции на околонулевых промежутках функции, умноженных на длинну этих промежутков.
Вот суть вопроса, если суммировать кучу значений квадрата получится куб делить на три. Почему? Можешь мне это пояснить без производных интеграла равных подинтегральной функции? Или это невозможно вывести по другому кроме как через производные и если я это не понимаю то стоит смириться и считать дальше не задумываясь о глубинной сути интеграла?
>>375251>>375293
#426 #375251
>>375250
Пусть f(x)=x^2, интеграл от 0 до 1 по определению - это lim(n->inf) sum[k=0..n-1] f(k/n) * 1/n (поделили промежуток [0..1] на n кусков длины 1/n и просуммировали их, домножив на значение функции в точке, а потом устремили 1/n к нулю). Если возьмёшь предел то получишь те самые 1/3 которые интеграл от 0 до 1 x^2 по dx. Вот и всё.
>>375255
#427 #375255
>>375251

>(поделили промежуток х=[0..1] на n кусков длины x^2/n и просуммировали их, домножив на значение функции в точке между разрывами кусков, а потом устремили x^2/n к нулю)


Правильно я понял?
>>375258
#428 #375258
>>375255
Ну если до x суммировать собрался, то да. У меня в случае x=1. Попробуй посчитать предел, кстати - это школьное упражнение не требующее никакого знания о производных даже.
>>375261>>375279
#429 #375261
>>375258
Я же написал

>промежуток х=[0..1]

>>375262
#430 #375262
>>375261
Ну это очень неаккуратная запись которая мне непонятна. У меня x - это число, а не промежуток.
#431 #375264
>>375222
На самом деле вполне годный метод проверки своих реальных познаний. Человек, который разбирается в математике, сможет обучить будущие поколения интегральчикам.
#432 #375279
>>375258
Я поделил промежуток на 10 частей и получил 0.275
Поделил на 20 и получил 0.369025
Считал вручную на бумажке, так что во втором случае у меня может быть ошибка. Или все так и задуманно?
>>375280
#433 #375280
>>375279
Не знаю, я поделил на 20 частей и получил 0.308
https://www.wolframalpha.com/input/?i=sum+(k/20)^2++1%2F20+where+k+from+0+to+19
поделил на 1000 частей, получил ~0.332
https://www.wolframalpha.com/input/?i=sum+(k/1000)^2+
+1%2F1000+where+k+from+0+to+999

что совпадает с той самой 1/3-0.333..., которая получается, если в x^3/3 подставить 1.
>>375281>>375283
#434 #375281
>>375280
Не совпадает, а даёт очень хорошее приближение.
#435 #375283
>>375280
Ну значит я где то в разрядах ошибся.
Но вообщем суть я понял, разбить на промежутки, найти значения на концах промежутков, умножить на длинну промежутка, суммировать.
Алсо, это все так и задуманно, чтобы подинтегральная функция была рвана призводной интеграла или так звезды встали?
>>375284
#436 #375284
>>375283
Ну я опять же отсылаю к статье на википедии. На ней написано примерно следующее: интеграл на очень маленьком промежутке [x0..x0+eps] например примерно равен eps f(x0). Иначе говоря F(x0+eps)-F(x0) ~ epsf(x0) -> (F(x0+eps)-F(x0))/eps ~ f(x0) если устремить eps к нулю получим F'(x0) = f(x0). Там ведь даже картинка есть, попробуй прочесть внимательно и вдумчиво раздел про geometric meaning.
>>375321>>375325
#437 #375293
>>375250

> длинну


Брат, прекрати пожалуйста.
>>375298
#438 #375298
>>375293
Дленну?
#439 #375321
>>375284
Ты ведь понимаешь что мне легче у тебя спросить чем читать!?
У меня очень плохо с английским, на русской викпедии пояснено слишком сложно
Я прочитал но не понял как он с производной скручен. Там поясняют как найти площадь красной полоски, и через нее выразить площадь синей области, но потом я теряю нить разговора и нахожу её только на словах

>Computing the derivative of a function and “finding the area” under its curve are "opposite" operations. This is the crux of the Fundamental Theorem of Calculus.

>>375327
#440 #375325
>>375284
ВОт давай я сейчас перескажу примерную суть из того что я там понял.

Есть другой способ найти площадь полоски. Как показано на рисунке ш умножается на функцию, из за чего находится площадь треугольника.

формула1

По факту уравнение станет точным если добавить к результату незакрашенную выделенную область.

формула2

переставляя члены получаем

формула3

При приближении ш к нулю, "лишняя" площадь приходящаяся на треугольник тоже стремится к нулю.
Это происходит из за того что "лишняя" площадь треугольник меньше или равна незакрашенной выделенной области. Более точно

формула4

Из за непрерывности функции последнее выражение тоже стремится к нулю при стремлении ш к нулю. Из за этого левая часть уравнения тоже стремится к нулю при стремлении ш к нулю

формула5

Получается что f(x) = A′(x). Так, производная площади существует так же как и сама функция и её значения. Так площадь функции является первообразной изначальной функции. Вычисление площади под кривой и нахождение производной являются противоположными операциями. Что и есть основной теоремой калькулуса.
>>375326>>375330
29 Кб, 720x371
#441 #375326
#442 #375327
>>375321

>У меня очень плохо с английским, на русской викпедии пояснено слишком сложно


Ты статью Fundamental theorem of calculus читаешь? Сегодня сам спрашивал в соседнем треде про первообразные, потом уже статью ту открыл. Я так понял, что связь с производной выходит их того, что h стремится к нулю, то есть по сути площадь бесконечно малого прямоугольника. А формула вот эта A(x+h)-A(x) /h это всё равно что для производной, только там буквы другие. Ну а она вышла из выражения для f(x), где делится всё на h.

>As h approaches 0 in the limit, the last fraction can be shown to go to zero.


Хз, зачем я написал, просто самому до конца хочется разобраться тоже.
>>375331
#443 #375330
>>375325
Там поясняют, что площадь красной полоски (A(x+h) - A(x)) делённая на длину основания (h) красной полоски примерна равна производной функции f в точке x, если h достаточно маленькая.
>>375325
Ну вроде хорошо перевёл. На каком месте ты теряешься?
>>375332>>375337
#444 #375331
>>375327
Да.
Я уже говорил о том что интеграл это сумма околонулевых производных, но мне сказали что это не так.
#445 #375332
>>375330
Ну так я же спрашивал, интеграл это куча производных, а ты сказал что нет.
>>375334
#446 #375334
>>375332
Ну так потому что действительно нет. Производная, если грубо, это f(x+h) - f(x) / h, а интеграл - это f(x) h, разве же это похожие штуки?
>>375341>>375342
#447 #375337
>>375330
Вот ты имеешь ввиду что сумма производных во всех точках интеграла равна внутриинтегральной функции.

При этом, площадь под кривой вычисляется по методу огромного числа прямоугольничков, сумма площади которых стремится к интегралу.

При этом, точное значение интеграла находится через операцию обратную производной и утверждается что эта операция и есть интегрирование.
#448 #375341
>>375334
Ну смотри, интеграл это f(x)h. Вернемся к квадратной функции.
Если дифференцировать, получим
(f((x+h)^2)h-f(x^2)h)/h=f(x+h)^2-f(x^2)=f(2hx), так?
>>375574
#449 #375342
>>375334
Но только из f(x)*h не вывести ничего. Или можно?
#450 #375410
>>375409
Привет, что здесь забыл?
#452 #375417
https://www.youtube.com/watch?v=Y0A6ac3BTIg
А вы ещё тут окрестность-ловушку бугуртите, вот это я понимаю строгое математическое объяснение.
62 Кб, 1427x379
#453 #375433
Читаю доказательство того, что если мы почленно проинтегрируем степенной ряд, полученный ряд будет иметь тот же радиус сходимости. Рассуждения пикрелейтед же совсем нестрогие. Откуда мы знаем, что если lim sup a_n = a и lim sup b_n = b, то lim sup (a_n ^ b_n) = a^b? Это же доказывать надо. Или, может, я че-то не понимаю и есть какое-то общее утверждение, из которого такие штуки следуют?
>>375444
#454 #375440
>>375412
Это шень что-ли?
#455 #375443
А вот я не понимаю, почему вообще не пишут полных доказательств, с каждым переходом? Вежь это нерационально, студенты потом тратят столько воемени, задавая одни и те же вопросы, что за него можно было бы все учебники по 10 раз переписать. Почему не сделать сразу нормально?
#456 #375444
>>375433
Причем

> если lim sup a_n = a и lim sup b_n = b, то lim sup (a_n ^ b_n) = a^b


это даже неверно. Возьмем a_n = e^{-n^2}, b_n = 1/n.
Тогда a_n -> 0, b_n -> 0, но a_n^{b_n} сходится не к 1, а к 0. Охуенное доказательство, че сказать.
#457 #375468
>>375443
С каждым переходом ты бы такое доказательство из какого-нибудь реального анализа за всю жизнь не прочёл.
Но если говорить просто о подробных доказательствах, то тут в силу вступают экономические причины - в книге нужно компактно уместить пакет тем, а не растянуть всё на две тысячи страниц, ведь последнюю книгу никто тогда не купит.
>>375487
#458 #375487
>>375468
Ну это ерунда какая-то. Давайте тогда вообще перестанем писать книги и ограничимся методичками на 50 листов. Никто не мешает взять и напечатать два тома: обычный учебник в одном, подробные доказательства в другом. Да и вообще, количество использованной бумаги уже давно не является экономической причиной в таких вещах. Короче - нет, вообще мимо.

Если уж говорить об экономике, то сказал бы лучше о временных затратах на написание этих доказатедьств. Но и это, на мой взгляд, не проблема.
>>375493>>375613
#459 #375493
>>375487

>ограничимся методичками на 50 листов


SUKAAAAAAAAAAAAAAAAA, как же мне бугуртно от этого. Как вспомню как учился по методичке на 50 листов так пдаю в обморок от бугурта
#460 #375503
>>375443
Мозг устроен сильно иначе, чем математика. Поэтому знание доказательств часто вообще ничем не помогает в понимании, а наоборот идеи, не имеющие отношения к математике, могут давать мощный толчок в понимании. Здесь я соглашусь в Ромой Михайловым, что имеют место некоторые "узоры", которые человек выцепляет из реальности. Эти узоры могут быть общими для многих явлений.

Поэтому когда доказательство несёт в себе некий важный узор, его стоит записывать. Если же это манипуляция терминами (просто техническое док-во, например), человек потратит время, но для себя ничего не усвоит.
>>375514
#461 #375514
>>375503
Роман, конечно, хороший человек, но ты сейчас ведешь себя как типичный русский - заметил? В ответ на вполне конкретный технический вопрос пускаешься в пространные философские рассуждения о познании и смысле жизни.

"А как вот из этого получилось вот это?" - типичный и до жути распространенный вопрос. Я не предлагаю обсуждать проблемы обучения и роль математики в формировании жизненной позиции поциентов. Я спрашиваю, почему нельзя один раз напечатать все переходы, чтоб потом студенты не задавали одни и те же вопросы преподам и форумам по тыще раз.

Я достаточно четко обозначил вопрос?
#462 #375516
>>375514

>но ты сейчас ведешь себя как типичный русский - заметил? В ответ на вполне конкретный технический вопрос пускаешься в пространные философские рассуждения о познании и смысле жизни.


O_O Что простите? Конкретно охренел с такого выпада.

> вот я не понимаю, почему вообще не пишут полных доказательств, с каждым переходом?


Ты сам свой вопрос-то видел?
И сразу кидаться в национальности. Как культурно-то и по-европейски.
>>375519
#463 #375517
>>375514
Не говоря уже о том, что на вопрос я ответил, и даже вынес это в отдельный абзац.

>Поэтому когда доказательство несёт в себе некий важный узор, его стоит записывать. Если же это манипуляция терминами (просто техническое док-во, например), человек потратит время, но для себя ничего не усвоит.


Читай если оно идейное, его записывают полностью, а техничные нерационально записывать.

В общем, как в говне искупался, немного противно, что я для такого человека вообще потратил время на ответ.
>>375519
#464 #375519
>>375516
Лол, и вот опять. Вряд ли кто-то еще кроме русских воспринимает как оскорбление констатацию факта принадлежности к своему народу. Грустно это, конечно, но совсем уж оффтоп.

В моем посте не было никаких выпадов, ты неверно воспринял его тон - который я даже и не думал хоть как-то приближать к оскорбительному. Перечитай, пожалуйста, мой пост с учетом этого.

>>375517
Так я же ответу именно на этот твой абзац посвятил и свой абзац тоже - тот, который начинается со слов "А как вот из этогго получилось вот это?".
>>375520
#465 #375520
>>375519
Как оскорбление я воспринял в первую очередь называние меня "поциентом". Впрочем, и то, что "типичный русский" для тебя оскорбление, сомнений особо не вызывает.
>>375526
#466 #375524
>>375514

>"А как вот из этого получилось вот это?"


Что докажет конкретный автор, зависит от его позиций исключительно. Есть и те, кто записывают все доказательства, включая совсем технические, мне, например, приходилось таких читать. В основном это считают нерациональным / ленятся и опускают детали, чтобы больше времени уделить более важным вещам.
>>375527
#467 #375526
>>375520
Гхм, я не называл тебя поциентом. Могу только еще раз посоветовать внимательно и с чистой головой прочитать мой пост, если это тебе интересно. Но конструктивного в нашем разговоре я вижу мало, так что вместо этого предлагаю лучше закончить его поскорей.

И вернуться к проблеме недостаточной строгости доказательств из учебников.
#468 #375527
>>375524
Так вопрос как раз в том, почему в основной своей массе люди считают это излишним и нерациональным, хотя есть некоторые доказательства обратного.

Как же мы будем все доказательства в какие-нибудь агды с коками переводить, когда у нас даже культуры просто подробных неформальных доказательств нет?
>>375530
#469 #375530
>>375527

>почему в основной своей массе люди считают это излишним и нерациональным, хотя есть некоторые доказательства обратного.


Можно пример какой-нибудь?

>Как же мы будем все доказательства в какие-нибудь агды с коками переводить


Так для этого всё равно слегка другие умения требуются, а не умение просто доказать теорему. Для этого стоит теорию типов в совершенстве знать, и базовые пакеты.
>>375532
#470 #375532
>>375530
Ну вон же выше анон спрашивал "как отсюда сюда перешли", например.
#471 #375574
>>375341
Бамп что ли. Я честно прочитал весь раздел про геометрикал мианинг и пол ночи уснуть не мог, думал где я ошибаюсь.
>>375575>>375576
#472 #375575
>>375574
Даже не так, я продифференцировал функцию интеграла, таки получил внутриинтегральную функцию по верхнему пределу, но так и не понял как у меня это получилось.
>>375576>>375766
#473 #375576
>>375575
>>375574
И да, я в поисках по интернету видел как к правильному выражению интеграла пришли с помощью, если я правильно понял, алгебры и чего то там про арифметические прогрессии.
На примере площади квадратичной функции от одного до двух там получили уравнение в духе 7/3-дробь_с_dx-дробь_с_dx^2. Собственно при приближении dx к нулю эти дроби становились КРАЙНЕ МАЛЫМИ и не влияли на результат. Вот про это судя по всему мне надо пояснить.
>>375766
#474 #375613
>>375487
Всё ложь.
>>375614
#475 #375614
>>375613
Скукожь ее, скукожь.
#476 #375618
>>372131 (OP)
Анон, существует ли последовательность рациональных чисел, сходящаяся к квадратному корню из 2? Если да, то какой она имеет вид?
>>375620
#477 #375620
>>375618
Нужно было задать в более общем виде: для любого ли иррационального числа существует последовательность рациональных чисел, сходящаяся к нему?
>>375624
#478 #375624
>>375620
О да. И даже бесконечно много их.
>>375626
#479 #375626
>>375624
Меня больше интересует вид хотя бы одной из таких.
>>375632>>375633
#480 #375630
>>375443
С какого-то момента дальнейшее уточнение доказательства только мешает его воспринимать. Представь себе как крайнюю точку этого пути настоящее формальное доказательство из логики на n страницах. Понять будет гораздо сложнее, чем придумать самому. На самом деле вообще невозможно, если заранее не знать.
Как менее крайнюю точку можешь представить учебник матана, где каждое преобразование выражения сопровождалось бы словами вроде "Мы можем перестановкой слагаемых x и y перейти от выражения A к выражению B из-за того, что вещественные числа образуют поле (теорема n), а поле обладает свойством коммутативности (аксиома m из определения)".
Поэтому нужна золотая середина. Каждый автор учебника видит её по-своему.
>>375631
#481 #375631
>>375630
Так ведь придумали же для этого иерархические доказательства, или как они там правильно называются.

Кроме того, я не предлагаю менять золотую середину - для этого у меня мозгов маловато и яйца недостаточно круты. Я предлагаю к учебнику добавить томик с заголовком "Доказательства", где все будет педантично и без пробелов.
>>375964
#482 #375632
>>375626
A_n = round(sqrt(2)*2^n)/2^n
round -- округление до целого (любое из).

A_n рациональное и отличается от sqrt(2) не более чем на 2/2^n.
>>375634
#484 #375634
>>375632
Ох, в общем, суть в том, чтобы представить все вещественные числа в виде суммы единиц с приравниванием к нулю. С иррациональными возникла проблема. Если рациональные представимы в виде суммы единиц, то нужно найти способ представить и иррациональные в таком виде.
То есть в характеристике последовательности не должно быть иррациональных чисел.
Тогда можно пойти по иному пути. Необходимо найти последовательность рац. чисел вида (m^2)/(n^2), сходящуюся к 2. Тогда последовательность корней этих чисел будет сходиться к sqrt(2). Есть идеи?
>>375635
#485 #375635
>>375634

>в виде суммы единиц с приравниванием к нулю


Чего??
>>375636
#486 #375636
>>375633
Спасибо, но там вроде только ряды, а не чистые последовательности. Но может и они сойдут.
>>375635
Последовательно можно показать, что любое рац число "порождается" единицей и неким характеристическим уравнением. В некоторых из них нужен еще ноль.
При чем, умножение даже в общем случае пока что вплоть до рац. чисел выражается через сумму этих самых единиц.
>>375644>>375645
#487 #375644
>>375636

>Спасибо, но там вроде только ряды, а не чистые последовательности.


Так последовательностью будут частичные суммы ряда.
1 Кб, 219x60
#488 #375645
>>375636

>Спасибо, но там вроде только ряды, а не чистые последовательности.


В той же статье есть https://en.wikipedia.org/wiki/Square_root_of_2#Computation_algorithms
Если a_0 > 0, a_n стремятся к корню из 2.
30 Кб, 1086x247
10 Кб, 1067x140
#489 #375668
Посаны, помогите разобраться с русской и английской терминологией. Пикрелейтед 2 (Львовский), по-моему, не согласуется с термином sequentially compact. В английской вики:

> Topological space is sequentially compact if every infinite sequence has a convergent subsequence.


> Topological space X is limit point compact if every infinite subset of X has a limit point in X.



Очевидно, что в метрическом пространстве sequential compactness эквивалетна limit point compactness. Как я понял, в неметризуемом пространстве это может быть не так.

Я правильно понимаю, что в русскоязычной литературе секвенциальная компактность - это на самом деле limit point compactness, а термина для sequential compactness нет вообще?
>>375701>>375736
#490 #375699
А кто сейчас в Бурбаки, кстати? Я что-то и не знаю видных франц. математиков, работающих на данный момент.
#491 #375701
>>375668
Эти понятия в любом топологическом пространстве эквиваленты.
>>375704
#492 #375704
>>375701
Sequentially compact --> Limit-point compact.

Limit point compact, first-countable and T1 --> Sequentially compact.

Наверное ты хотел сказать в любом метризуемом?
>>375719>>375733
#493 #375719
>>375704
Ну не в метризуемом обязательно, просто в любом человеческом.
С этим вот короче

> first-countable and T1


Я, правда, не понял зачем там счётная база.
#494 #375733
>>375704
Ну и говнище! Ну и жопа! Нахрена нужны эти патологические топ. пространства, а? Зачем? Т1 блджадт, счётная база... говно.
#495 #375736
>>375668

>Я правильно понимаю, что в русскоязычной литературе секвенциальная компактность - это на самом деле limit point compactness, а термина для sequential compactness нет вообще?



Нет sequentially compact и limit point compact - это разные свойства.
>>375817
#496 #375766
>>375575
>>375576
Разобрался? Сегодня читал отсюда статью Integrals, первые главы. Там на примере суммы и разности вроде поясняют. Но я бегло прочел, тебе, возможно, поможет.
>>375767
#498 #375817
>>375736

> sequentially compact и limit point compact - это разные свойства.


Это я понимаю. Я хотел прояснить как в русскоязычной литературе что называется. Но чет уже интерес пропал.
#499 #375835
чего-то в этом году без гробов обошлось
https://www.imo-official.org/year_individual_r.aspx?year=2016&column=total&order=desc
>>375836
#500 #375836
>>375835
Почему большинство участников в топе из азиатских стран? Типа там все настолько плохо с наукой, что умным людям не остается ничего кроме как олимпиадки дрочить?
#501 #375838
>>375836
Потому что их много.
#502 #375840
>>375836
У азиатов и IQ выше в среднем, например.

>настолько плохо с наукой, что умным людям не остается ничего кроме как олимпиадки дрочит


Это олимпиада школьников, дурик.
#503 #375841
https://www.imo-official.org/year_country_r.aspx?year=2016
Хаха СЕВЕРНАЯ КОРЕЯ выше РОССИИ.
2 Кб, 113x135
#504 #375842
>>375836
Радует, что все 4 человека в топе команды США -- тоже азиаты))
Allen Liu, Yuan Yao, Junyao Peng, Ashwin Sah
>>375848
#505 #375848
>>375842
Что с командой Лаоса? Почему у них все по нулям? Но при этом рейтинг указан выше, чем на предпоследнем месте.
#506 #375964
>>375631

>Так ведь придумали же для этого иерархические доказательства, или как они там правильно называются.


К сожалению, без понятия. Про что это?

>где все будет педантично и без пробелов.


Насколько педантично и без пробелов? Аргументы вновь в силе. Настоящее формальное доказательство будет нечитабельно, прочитает его ровно 0 человек. Почти формальное будет почти нечитабельно и прочитает его парочка мазохистов.
В итоге окажется, что правильное название для этого томика будет "Чуть более подробные доказательства".
>>375965
#507 #375965
>>375964

> Про что это


Все доказательство в виде серии утверждений, каждое из которых можно развернуть (как в проводнике), чтобы увидеть переход уровнем ниже. То есть когда все закрыто, есть только предпосылка и вывод. Когда все развернуто, полное формальное докво. Если какой-то переход не понятен, разворачиваешь, все еще непонятно - развоачиваешь дальше, етк.
#508 #375970
>>375836
Потому что в азиатских странах мучить школьников считается нормальным (причем, вне зависимости от уровня демократии). В России тоже принято, но не в такой степени. В Китае школьников в рамках подготовки натурально на три месяца закрывают в интернате, где они задрачивают все, что можно. Поэтому практически любую геометрию они без проблем посчитают. Даже если это займет три листа.
#509 #376012
Пересоздайте тред с картинкой, где аниме квантор всеобщности держит. Заебали своими олимпиадами.
>>376020
Перекатs #510 #376018
241 Кб, 1920x1080
#511 #376020
>>376012
Она в прошлом треде была.
Тред утонул или удален.
Это копия, сохраненная 3 ноября 2016 года.

Скачать тред: только с превью, с превью и прикрепленными файлами.
Второй вариант может долго скачиваться. Файлы будут только в живых или недавно утонувших тредах. Подробнее

Если вам полезен архив М.Двача, пожертвуйте на оплату сервера.
« /sci/В начало тредаВеб-версияНастройки
/a//b//mu//s//vg/Все доски